Sunteți pe pagina 1din 82

Question

Q1. The concept of indemnity is based on the key principle that Correct
policyholders should be prevented from Answer
1. Insuring existing losses. 4
2. Making false insurance claims.
3. Paying excessively for insurance cover.
4. Profiting from insurance.

Q2. If 5% bonus is given every year then for a SA of 1 lakh,


what will be the payout after 15 years in a simple 2
revisionary bonus system?
1. 60000
2. 7500
0
3.10000
0 4.
2
5000
Q3. The business of Insurance is connected with ...................
1. physical values of assets
2. economic values of assets
3. metaphysical values of assets
4. market values of assets

Q4. Mr. shailsh has an endowment policy with 30 years policy


term, he has paid for eight years. The sum assured
Rs.8,00,000/- and accumulated bonus Rs.60,000/-. What is 1
the paid up value if bonus accumulated and if not bonus
accumulated?
1. 213333 & 273333
2. 213333 & 229333
3. 229333 & 273333
4. 229333 & 293333

Q5. Under the IRDA guidelines for Claim settlement, any 3


queries or additional documents can asked from the
claimant within
1. 5 days from the receipt
2. 10 days from the receipt
3. 15 days from the receipt
4. 20 days from the receipt
2
Q6. Ombudsman has to give his decision within how many days?
1. 15 Days
2. 30 Days
3. 2 Months
4. 3 Months
2
Q7. Pooling of risk in insurance means
1. The premium collected & deposited in a pool
2. All similar risks are pooled together
3. Premium is pool to make claims
4. Contribution of insurance company

Q8. If the annual premium for a plan is 32000 and a frequency 2


loading of 4% is added in a quarterly premium what is
the amount that needs to be paid.
1. 8000
2. 8320
3. 9456
4. 9240

Q9. What are the factors involved in calculating Surrender


Value of the Policy?
1. Number of year's premium paid. 4

1
2. Number of premiums payable.
3. Sum Assured
4. All of the above.
Q10. Mr. Rajesh has taken policy from ABC insurance company for Rs.
500000 Sum Assured by paying Rs. 50,000 premium per year. 2
Company declared 5% Simple Reversionary bonus, what is
bonus amount?
1. Rs. 2500/
2. Rs. 25000/
3. Rs. 250000/
4. Rs. 5000/-

Q11. In an insurance contract 'consideraton' means ........ 3


1. proposal form
2. Advisors confidential report
3. premium
4. claim

Q12. Where do you not find insurable interest in the following 4


options .......
1. surety-co surety
2. employee-employer
3. husband-wife
4. brother-sister

Q13. Which of the following statements in correct in connection 1


with assignment?
1. Assignee cannot make fresh nomination in the policy
2. The assignor need not be major at the time of assignment.
3. Section 45 of the insurance act speaks about assignment.
4. Conditional assignment and absolute assignment are one and the
same.
Q14. Life Insurance also known as: 1
1. Value Contract.
2. Indemnity
3. Commercial Contract
4. Speculative

Q15. Gaurav is working in MNC at the age of 32 bought an


Endowment Plan. He had nominated his 1 year old daughter
Saanvi, but not able to get the Signature of her appointee 2
due to unavailability of his spouse .after 5 year. He died in road
accident, now claim money would be payable to:
1. Nominee only
2. Legal heir of the life assured
3. Appointee Only
4. will be Payable to Saanvi (Nominee) at the age of 18th

Q16. Agent will be called as ..... 1


1. Primary Underwriter
2. Main Underwriter
3. Chief Underwriter.
4. Information Underwriter.

Q17. When an illiterate person wants to have a policy ..... 1


1. an impression of the left thumb is taken and third party has to
attest it
2. an impression of the left thumb is taken and the advisor has to
attest it
3. an impression of the left thumb is sufficient and need not be
attested

2
4. A relative of the illiterate person has to sign on behalf of that
illiterate person.
Q18. Mr.Feroz has applied for an insurance cover of Rs.4 crores. The
Company will accept or reject the proposal only after
confirming from one of the following agencies.
1. Financial Inspection agencies
2. Specialized inspection agencies
3. Credit worth inspection agencies
4. Insurance Investigation Agencies

Q19. Ram Lal is an insurance policyholder. He has recently shifted his


home from New Delhi to Noida. He wants the address to be
changed. This change in policy document will be effective
through?
1. Terms & Condition.
2. Preamble.
3. Endorsement.
4. Schedule.

Q20. In which of the following plan remaining part of the Sum


Assured is paid on maturity?
1. Endowment Plan
2. Convertible Plan
3. Money Back plan
4. Term Plan

Q21. The income of an individual can be protected with the help of


1. a unit linked policy
2. a term life policy
3. an endowment policy
4. a money back policy

Q22. In case of life insurance, Insurable interest must exist


1. At inception of policy
2. Not needed
3. At the time of claim
4. Any time during the contract

Q23. A customer surrenders his policy on Feb 2010 As per Agents code
of conduct, an agent can get a new policy from this customer from
Feb which year
1. 2011
2. 2012
3. 2013
4. 2014
Q24. Rakesh wants to buy a policy primarily for Risk Cover but at the
end of the term he wants to get at least some return. Under which
policy he will get these benefits
1. Endowment plan
2. ROP plan
3. Whole life plan
4. Ulip

Q25. Bank interest is accumulated


1. Monthly
2. Yearly
3. Quarterly
4. Once in 6 months

Q26. Mr. Rajgopal has invested some money. He has been

3
informed clearly about the tenure, interest rate and method of
payment of interest at the inception of the investment itself. His
investment may be in.....................
1. Life Insurance
2. Mutual fund
3. Shares
4. Bank deposits

Q27. An elderly person wants to use tax efficient investment and


4
invests in senior citizen saving scheme. Its the impact in his
taxation
1. He will get tax benefit up to 5000
2. He will get tax benefit up to 25%
3. He will get a reduction in tax slab
4. His investment would be deducted from taxable income

Q28. Some Health Plans will give coverage for family members 3
also. We will call these plans as ....
1. Family Health Insurance Plans
2. Total Protection Policies.
3. Family Floater Health Insurance Plans.
4. Family Rakshak Health Plans.

Q29. While calculating the expected returns from investments and 3


savings, an individual should make provisions for
1. Taxation only.
2. Inflation only.
3. Taxation & inflation both.
4. No provision is require4.

Q30. The life expectancy in India is constantly improving and is well


above 60 years. This also brings along challenges. 3
These challenges can be covered through
1. Equity.
2. Bank fixed deposits.
3. Health insurance plan.
4. Savings insurance plan.

Q31. If a person is concentrating more on health plan and 4


retirement plan then at which stage of life is he in.
1. Young unmarried
2. Young married
3. Young married with Kids
4. Pre retirement

Q32. Manish and Manisha is a married couple with one child. They
want to plan for savings, child education/marriage and their 1
retirement and protection of income. Which should be their
lowest priority?
1. Marriage.
2. Savings.
3. Education.
4. Protection.

Q33. Mr. Ramachandra's son Mr. Bharat has just employed as a


software engineer, daughter Anusha has got married. Now 1
Mr. Ramachandra is free from his burden. So Mr.
Ramachandra is now in the .................. stage.
1. Pre-retirement stage
2. Post retirement stage
3. Married with older children stage
4. Employment stage

4
Q34. During the recommendation stage the advisor needs to ......
1. recommend the products that best meet the client's needs
2. recommend to wait some days to invest
3. recommend the new product the company has recently
launched
4. Recommend to take his own time to take a decision.

Q35. During the fact finding session as an agent you should follow
which one of the following process?
1. Prioritize, Identify & Quantify Needs.
2. Identify, Quantify & Prioritize Needs.
3. Quantify, Identify & Prioritize Needs.
4. Prioritize, Quantify & Identify Needs.

Q36. The consequences of these risks which will affect specific


individuals or local communities in nature is called as
1. Pure risk
2. Financial risk
3. Particular risk
4. Physical hazard

Q37. Agent who is a licensed intermediary is actually is ?


1. A legal person to act on behalf of the re-insurer
2. B.A legal person to act on behalf of the insurer
3. C.A legal person to act on behalf of the contract
4. D. An authorized agent to act on behalf of the legal company

Q38. When client declines the recommendation by the insurance


adviser even after resolving the concerns, the adviser should
1. Persuade the client to purchase the policy.
2. Should ask for reference who might be interested in financial
planning.
3. Should never approach the client ever again.
4. Should do nothing.

Q39. What key benefit high persistency ratios have on insurance


adviser?
1. High renewal income.
2. Less renewal income.
3. Fewer client bases.
4. Unsatisfied clients.

Q40. Who is a regulator, supervisor and monetary authority of the


financial system in India?
1. IRDA
2. SEBI
3. RBI
4. SBI
Q41. For the customer whose claim has been refused by the
company, the three places where he/she should follow up are -
Ombudsman, IRDA Customer Grievance Cell and
1. Consumer Forum
2. COPA
3. Sebi
4. Life Insurance Council
Q42. What is meant by a claim under insurance policy?
1. A demand to fulfill the policyholder's obligations.
2. A demand to fulfill the insurer's obligations.
3. Any demand made by the policyholder on the insurer.
4. All of the above.

5
Q43. Mr.Sampath has taken a policy on January 10th 2006 for a term
of 12 years. Due to financial instability he discontinued to pay
the premium from 201 He died on August 13th 201 What will
happen to his death claim ?
1. Eligible for claim as the premium was paid from 2006 to Dec 2
2010.
2. Not eligible for Claim settlement
3. Claim will be settled after deducting the unpaid premium
4. Claim will be settled after the deduction of unpaid loan amount

Q44. Indisputability clause can be enforced by the insurance company 2


during the
1. First five years of policy
2. First Two years of policy
3. Claim
4. Inception of the policy

Q45. The foreign direct Investment (FDI) in Indian Insurance 3


Industry is allowed up to ...
1. 0.5
2. 0.25
3. 0.26
4. 0.27
Q46. The Insurance Institute of India (III) was formed in 3
1. 1956
2. 1999
3. 1955
4. 1947
Q47. What is the minimum Sum Assured allowed for Micro 1
Insurance?
1. 5000
2.10000
3.15000
4. 20000
Q48. How Many offices of Ombudsmen the Government body has set 2
up to Resolving insured customer's grievances.
1. 15
2. 12
3. 14
4. 16
Q49. Which of the following is a Non Standard Age Proof 3
1. PAN Card
2. Certificate of Baptism
3. Certificate from Village Panchayat
4. Certificate from School or College

Q50. Ethics can be defined as... 4


1. Those values we commonly hold to be "good" and "right".
2. Behavior that is based upon the moral judgments of an
individual
3. A study of what makes one's own actions right or wrong.
4. All of the above.

6
Questions Correct Answer
Q1. The concept of indemnity is based on the key principle that 4
policyholders should be prevented from
1. Insuring existing losses.
2. Making false insurance claims.
3. Paying excessively for insurance cover.
4. Profiting from insurance.

Q2. For assessing the risk of a group health insurance policy, which of the 3
following information is the most critical
1. Group lifestyle
2. Employees
3. Age of the group
4. Medical history of group

Q3. In Insurance terms, pooling of risk is 4


1. Using the same pool for paying claims of car & life insurance.
2. Using different pool for paying claims of life insurance.
3. Using the same pool for paying claims of life & house insurance.
4. Using the same pool for paying claims of life insurance.

Q4. Suresh is suffering from Asthma and the policy is been done on joint
life basis and the need for nomination under the plan will be as 1
1. Joint life policy
2. Not Possible
3. 2.11805555555556
4. Not Applicable

Q5. Health insurance rider and critical illness rider in classified under 2
1. Life and non life respectively
2. Both life insurance
3. Both non life insurance
4. Nonlife and life respectively.

Q6. Ombudsman has to give his decision within how many days?
1. 15 Days 2
2. 30 Days
3. 2 Months
4. 3 Months

Q7. Both the parties to a contract must agree and understand the same thing
and in the same sense which is called
1. Consideration. 3
2. Legality of an object.
3. Consensus ad idem.
4. Acceptance.

Q8. Ravi was expecting a claim amount of Rs. 12, 00,000 from insurer. But
it was rejected. He feels that it is repudiated on wrong reasons. Which 2
consumer forum can he approach?
1. National Commission
2. District Level
3. State Level
4. Mandal Level

Q9. Total annual premium Rs 32000 quarterly loading done 4 %. 3


Hence actual quarterly premium will be
1. 7680
2.9320
3.8320
4. 6600
Q10. Which of the following statements in correct in connection with
assignment? 1
1. Assignee cannot make fresh nomination in the policy
2. The assignor need not be major at the time of assignment.
3. Section 45 of the insurance act speaks about assignment.
4. Conditional assignment and absolute assignment are one and the
same.
Q11. Mr. Shanth has taken an endowment policy of 20 years. He has paid
premium for 10 years and now the policy is in force. At this point of 2
time can Shanth take loan?
1. Mr. Shanth will not be granted any loan
2. Mr. Shanth can take loan which should be certain percentage of the
surrender value of the policy.
3. There is no concept of loan in insurance policy
4. loans are allowed only in term plans

Q12. Mr. Josh was filling the proposal form but as his mother was sitting
beside him, even though he drinks and smokes he ticked "NO" in 2
smoking & drinking column of proposal form. This indicates ?
1. He has breached the non-disclosure of the fact
2. He has breached the company by concealing the facts
3. He has breached the company by fraudulent information
4. He had done an innocent misrepresentation

Q13. How assignment distinguishes itself from nomination? 1


1. Nomination does not transfer the title while assignment does.
2. Nomination transfers the title while assignment does not.
3. Nomination is made after policy is issued while assignment is done
before it is issued.
4. Nomination need not be informed but assignment needs to be
informe4.
Q14. Which clause lays down the mutual obligation of the parties regarding,
Payment of Premium by Life assured & payment of Sum Assured by 2
1. Lien Clause
2. Opertaive Clause
3. Proviso Clause
4. Schedule of Policy

Q15. Gaurav is working in MNC at the age of 32 bought an


Endowment Plan. He had nominated his 1 year old daughter Saanvi,
but not able to get the Signature of her appointee due to unavailability
of his spouse .after 5 year. He died in road accident, now claim money
would be payable to:
1. Nominee only
2. Legal heir of the life assured 2
3. Appointee Only
4. will be Payable to Saanvi (Nominee) at the age of 18th
Q16. In a case, the underwriter felt that the risk associated with
the person would decrease with time, then he would accept 2
the case with
1. A clause
2. A Lien
3. A Loading
4. Level Premium

Q17. Age proof submitted from Village Panchayat is: 2


1. Standard and accepted
2. Not standard but accept
3. Not at all accepted
4. Accepted with SSLC book/mark list

Q18. Moral Hazard reflects the ..............


...................................................... 1
1. Intentions and attitude of Proposer.
2. Habits and Hobbies of Proposer.
3. Occupation and Residence of Proposer.
4. Medical and Personal History of Proposer.

Q19. To explain the benefits of a product, the insurance adviser should


1. Provide the product brochure to the client.
2. Provide Benefit illustration documents to client.
3. Provide the terms and conditions document to the client.
4. Provide the website address to the client.

Q20. The two basic elements of most life insurance plans are 3
1. Guaranteed Benefit and Non-guaranteed Benefit.
2. Interest Benefit and Bonus Benefit.
3. Death Benefit and Maturity Benefit.
4. Bonus Benefit and Tax Benefit.

Q21. Mr. Shanth has got a pure endowment policy for 30 years for ^ the sum assured of
Rs. 75,000,00/-. It can be paid to him......................................................
1. when he dies
2. when he survives the term
3. when he is hospitalized
4. when he loses his job

Q22. An investor holds a wide range of shares. If the Reserve Bank


of India announces a series of significant interest rate 2
increases, the prices of these shares are most likely to
1. Become volatile
2. Decrease
3. Increase
4. Remain unchanged

Q23. Mr. Denny is married and has 2 children and his parents are
alive. He has taken a family floater plan. Under the plan who 4
all will be covered.
1. Denny
2. Denny and his wife
3. Denny, his wife and children
4. Denny, his wife, his children and his parents
Q24. Bank interest is accumulated

2
1. Monthly
2. Yearly
3. Quarterly
4. Once in 6 months
Q25. Mr. Rajgopal has invested some money. He has been informed clearly
about the tenure, interest rate and method of payment of interest at the
4
inception of the investment itself. His
investment may be in ...........
1. Life Insurance
2. Mutual fund
3. Shares
4. Bank deposits

Q26. Mukesh buys shares at lower price and sold at higher price, 2
the Difference between the two prices is known as:
2

1. Dividend Income
2. Captial Appreciation
3. Bonus Share
4. Interest on share

Q27. A lump sum withdrawal allowed as commutation in pension


1
plans is
1. 1/3rd of the accumulation fund
2. 1/4th of the accumulation fund
3. 1/2nd of the accumulation fund
4. Full withdrawal is allowed

Q28. Venu, aged, 32, had a Insurance cover of 10, 00,000/- . He was
approached by an advisor who made some analysis and told him that
his Insurance need is higher than 10, 00,000 1
and suggested that he surrenders the existing policy and buys a new one. This is an
example of
1. Churning
2. Proposing
3. Underwriting
4. Switching

Q29. Mr. Ranga has taken Critical Illness Rider. In which of the
following scenarios the insurance company will pay him ....................
.............................................................................................................. 2
1. When Mr. Ranga dies due to critical illness
2. When Mr. Ranga is diagnosed a critical illness
3. When Mr. Ranga's family member will be in critical illness
4. When Mr. Ranga dies due to severe road accident.

Q30. Kajal is investing for the purpose of retirement at the time of


vesting Age before receiving the regular annuity she has 2
option to withdraw/commute Upto:
1. 2/3rd of Accumulated Fund
2. 1/2th of Accumulated Fund
3. 1/3rd of Accumulated Fund
4. 1/4th of Accumulated Fund
Q31. Health insurance policy holder takes treatment in a hospital
which doesn't have cashless facility. How the policy holder 1
will get benefited
1. Need to spend and Get claim from the insurer
2. Need to add that hospital as TPA
3. Need not to claim
4. Need to switch to the another insurer

Q32. Perceived needs are those.... 3


1. Short term needs.
2. Imagined to be important by Advisor.
3. Imagined to be important by client.
4. Long term needs.

Q33. Need analysis involves identifying ..................


........................................................................1
1. Financial provision to meet predictable and unpredictable needs.
2. Capacity to pay the premiums in future.
3. Existing insurance plans.
4. Family and Employment details.

Q34. Mr.Mohith, an agent was explaining to his customer the


guaranteed benefits of the product. Which one of the ^
following documents he would have used to explain the
3

same?
1. Fact Finding document
2. B.KYC document
3. Benefit illustration document
4. Client benefit document

Q35. Which one of the following documents distinguishes between Guaranteed and non-
guaranteed benefits?
1. KYC documents
2. Benefit illustration documents
3. Fact-finding sheet
4. Financial details

Q36. If the client does not wish to proceed with the 2


recommendations right at the moment the agent should
1. Insist on taking the product right away
2. Should ask for the reason for not going with the recommendation
3. Should ask for a future date from the client
4. Should review once again

Q37. The consequences of these risks which will affect specific 3


individuals or local communities in nature is called as
1. Pure risk
2. Financial risk
3. Particular risk
4. Physical hazard

Q38. Naresh is married and his daughter Sneha is 3 years old. Which 1
plan can he take?
1. Term & Children Plan
2. Annuity Plan
3. Whole Life Plan
4. Health Plan

2
Q39. Satish as an insurance advisor while recommending to his client Ramesh is not
suitable policies -
1. Satish should check the Ramesh 's commitment to the needs
2. Satish should outline the reasons for recommendation for a
particular policy
3. Satish should check the acceptance or rejection of the
recommendation.
4. Should advised him to surrender all the polices which are not buy
from satish.
Q40. An indemnity bond was signed in a policy when the claim was paid. This indicates
that the policy was.
1. It was lost.
2. It was paid up.
3. It was lapsed.
4. It was surrendered.

Q41. Generally insurance companies do not hold the premium in case of a fraud or
misrepresentation. However, due to which of the following circumstances the
insurer can retain the premium of the policyholder
1. Fraudulent claim
2. Indisputability clause
3. Redressal procedure
4. Pending decision from Ombudsman

Q42. Insurer will not pay the claim unless 3


1. The policyholder makes a demand.
2. The nominee makes a demand.
3. The event insured against happens.
4. The insurer completes the enquiry.

Q43. Which one of the following report is not considered in case of


Pankaj Bought a plan in 2009 & died in road accident in 2011 2
1. Policy first information report.
2. Advisor confidential report
3. Post Mortem Report.
4. Coroners Report

Q44. Which is the 2nd stage in Money Laundering? 1


1. Layering
2. Placement
3. Integration
4. Amalgamation.

Q45. What is the maximum stake can foreign company has in one insurance company in
India?
1. 0.74
2. 0.26
3. 0.24
4. 0.76
Q46. Which organization was formed with purpose to promote ^
insurance education and training in India?
1. Institute of Actuaries of Indil.
2. National Insurance Academy.
3. Chartered Insurance Institute.
4. Insurance Institute of India.
Q47. What is the disadvantage to the insurer in case the persistency falls
1. Lower profits
2. Higher profits
3. Increased Liability
4. Decreased Life fund

Q48. Where would you have to go if you wanted to buy a Kisan Vikas Patra?
1. Any Nationalized Bank
2. Insurance Company
3. Post Office
4. Mutual Fund Company

Q49. When can an insurance company give more than 35% first
1
year commission?
1. When the insurance company is in the first 10 years of operation
2. If the agent has worked with the company for more than 5 years
3. If the agent has worked with the company for more than 10 years
4. If the agent is doing more than 3 policies in a month.

Q50. Rahul is a licensed insurance agent. As agent he must carry ^


3
out his role in accordance with
1. Company's code of conduct.
2. IRDA Acts code of conduct.
3. IRDA Regulations code of conduct.
4. Insurance Acts code of conduct.

4
Question Correct Answer
Q1. Which of the following do not include the channel of indirect 3
marketing?
1. individual agents
2. bancassurance
3. insurance brokers
4. through internet

Q2. In Insurance terms, pooling of risk is 4


1. Using the same pool for paying claims of car & life insurance.
2. Using different pool for paying claims of life insurance.
3. Using the same pool for paying claims of life & house insurance.
4. Using the same pool for paying claims of life insurance.

Q3. Amit is looking for term insurance plan for protection of his
2
family, he is advised to approach to:
1. Property Insurance
2. Life Insurance
3. Health Insurance
4. Liabilty Insurance

Q4. According to insurance terminology which of the following is 2


correct?
1. Lung cancer is a hazard whereas smoking is a peril
2. Smoking is a hazard and lung cancer is a peril
3. Lung cancer is a peril and smoking is a moral hazard
4. Smoking is a moral hazard whereas lung cancer is a peril.

Q5. Under the IRDA guidelines for Claim settlement, any queries 3
or additional documents can asked from the claimant within
1. 5 days from the receipt
2. 10 days from the receipt
3. 15 days from the receipt
4. 20 days from the receipt

Q6. Which of the following can be an example of moral hazard? 3


1. a family history of heart disease
2. a person working in a chemical factory
3. a person consuming alcohol
4. A teacher working in a primary school.

Q7. Life insurance is the most important for which age group 2
1. Young
2. Pre- retirement
3. Retirement
4. children

Q8. If the annual premium for a plan is 32000 and a frequency


loading of 4% is added in a quarterly premium what is the amount
that needs to be paid.
1. 8000 2
2. 8320
3. 9456
4. 9240
Q9. If bonus is given under a plan the additional premium added
is known as
1. Loading
2. Investment 1
3. Frequency
4. Interest

Q10. Mr. Rajesh has taken policy from ABC insurance company for
Rs. 500000 Sum Assured by paying Rs. 50,000 premium per year. 2
Company declared 5% Simple Reversionary bonus, what is bonus
amount?
1. Rs. 2500/
2. Rs. 25000/
3. Rs. 250000/
4. Rs. 5000/-

Q11. In an insurance contract the insurable interest needs to be at the time of 3


1. claim
2. revival
3. inception
4. surrender

Q12. In an insurance contract 'consideraton' means ....................... 3


1. proposal form
2. Advisors confidential report
3. premium
4. claim

Q13. The principle of utmost good faith applies to ........................ 3


1. only insurers
2. only proposer
3. both insurers and proposer
4. neither insures nor proposer

Q14. Shamsher has a health insurance policy of ' 1, 00,000


individually and from his company for ' 2, 00,000. He falls sick and
got hospitalized. His hospital bill ran to ' 50,000. He claimed this 1
amount from his individual policy. Also, he placed the request with his
company for group policy claim, which was rejected. The reason for
rejection is.
1. Indemnity contract.
2. Value contract.
3. Deemed contract.
4. Rolling contract.

Q15. Rakesh has bought an Endowment, Money back, Term &


Annuity Plan he would like to avail Loan from:
1. Endowment Plan
2. Term Plan
3. Money Back Plan
4. Annuity Plan
1
Q16. What is the limit of tax benefit that can be availed of under
4
Section 80C?
50,000/
1. 15,000/
2. 10,000/
3. 100,000/Which of the following information does not appear in the First
Q17. Premium Receipt? 2
1. method and frequency of premium payment
2. Date of commencement of last premium
3. Date the policy matures
4. Date the last premium will be paid
Q18. Mr.Feroz has applied for an insurance cover of Rs.4 crores.
The Company will accept or reject the proposal only after 4
confirming from one of the following agencies.
1. Financial Inspection agencies
2. Specialized inspection agencies
3. Credit worth inspection agencies
4. Insurance Investigation Agencies
Q19. Rakesh purchased a life insurance policy. While writing a proposal form he hide that he
practices mountaineering.
Sadly he died in an accident while climbing Mount Everest. 4
The insurers rejected the claim.What is the reason for
rejection?
1. Innocent misrepresentation.
2. Fraudulent misrepresentation.
3. Concealment.
4. Non-Disclosure.
Q20. The client of reinsurer are 1
1. Insurance companies
2. Banks
3. Asset Management Companies
4. Brokers

Q21. Payment/Investments in Kishan Vikash Patra under post


3
office schemes is done
1. Regularly with no fixed term
2. Lump Sum with no fixed term
3. Lump Sum for fixed period of time
4. Regularly for fixed period of time
Q22. What is the advantage of converting physical gold assets to
1
gold ETFs.
1. Liquidity
2. More gold in value
3. Purity
4. More conversion value
Q23. After maturity In a Unit Linked Life Insurance Policy, customer does not get
received Maturity in a lump sum.

2
2
What is the possibility of receiving it in installments if it is
not a annuity plan
1. He has switched his fund
2. He has opted for Settlement option
3. He has he has redirected his past premium.
4. Policy was lapsed on the time of maturity
Q24. A customer surrenders his policy on Feb 2010 As per Agents
code of conduct, an agent can get a new policy from this 1
customer from Feb which year
1. 2011
2. 2012
3. 2013
4. 2014
Q25. Rakesh wants to buy a policy primarily for Risk Cover but at the end
of the term he wants to get at least some return. Under which policy 2
he will get these benefits
1. Endowment plan
2. ROP plan
3. Whole life plan
4. Ulip
Q26. The Central Bank has recently announced the decrease in interest 2
rates. The prices of bonds are
1. Likely to increase.
2. Likely to decrease.
3. Will fluctuate.
4. No change is likely.
Q27. In Cumulative bank deposit the interest that in normally compounded 2
on what basis.
1. Monthly
2. Quarterly
3. Half Yearly
4. Annually
Q28. In case of a term plan the maximum premium of the accidental rider 3
can be.
1. 100% of basic premium.
2. 50% of basic premium.
3. 30% of basic premium.
4. 35% of basic premium.
Q29. Ajay bought a share for Rs.110 and he sold when it was Rs.630.What 1
had happened to his share?
1. Capital Appreciation
2. Capital Profit
3. Capital Benefit
4. Capital Variation
Q30. While calculating the expected returns from investments and savings,
3
an individual should make provisions for
1. Taxation only.
2. Inflation only.
3. Taxation & inflation both.
4. No provision is require4.
Q31. In the context of financial planning, how is the difference between real 2
needs and perceived needs best described?
1. Real needs are financial needs and perceived needs are non-
financial needs.
2. Real needs are actual needs and perceived needs are based on a
client's thoughts and desires.
3. Real needs are identified by the insurance agent and perceived
needs are identified by the client.
4. Real needs are needs which satisfy an objective and perceived
needs are needs which do not satisfy an objective.
Q32. Perceived needs are those.... 3
1. Short term needs.
2. Imagined to be important by Advisor.
3. Imagined to be important by client.
4. Long term needs.
Q33. What do you mean by Double Income Family? 2
1. When a person of the family is involved in double businesses or
jobs.
2. When both the life partners are earning members of the family.
3. When the income of a family is just the double of income of
another family.
4. When the members of a family do not work at all.
Q34. Anand received post taxation 5% return on his fixed deposit in a bank. 2
If his net return is 3%, what can be the reason
1. Administrative charge
2. Inflation
3. Interest rate
4. Market Risk
Q35. In which of the following an agent will collect the customer's Personal 1
data, professional data and financial data
1. Fact Finding Sheet
2. Advisor Confidential Sheet
3. Customer Data sheet
4. Personal and financial data sheet
Q36. What is the stipulated time frame within which an insurer is supposed
to respond after receiving any communication from its policyholders?
1. 24 hours
2. 1 day 3
3. 10 days
4. 30 days
Q37. What is the factor which has influence on persistency?

4
1. Role of Agent
2. Product Design.
3. Policy Servicing
4. All of the above.
Q38. Agent who is a licensed intermediary is actually is ?

2
1. A legal person to act on behalf of the re-insurer
2. B.A legal person to act on behalf of the insurer
3. C.A legal person to act on behalf of the contract
Q39. 4. D. An authorized agent to act on behalf of the legal company
Why a building long term relationship with clients is
3
necessary?
1. A satisfied client may be the source of other potential clients
2. Reviewing financial needs and plans are necessary with changes
3. Agent has the option to recommending highest commission at any time.
4. Introduction of a new insurance product or a change in a client's Circumstance
Q40. Which of the following falls under voidable contract?

1
1. Misrepresentation
2. Lack of insurable interest
3. Fraud
4. None of the above

Q41. An individual is said to be competent to enter into a contract if they 1


are
1. 18 year old
2. 21 year old
3. 23 year old
4. 25 year old

Q42. In insurance, if a person is not heard for .................... years his is 3


presumed to be dead.
1. 5 years
2. 6 years
3. 7 years
4. 8 years

Q43. Pranab is not able to pay premium for continue his policy due
to some Financial problem, so he rather than surrendering 1
the policy, what Option he has to convert ....................... ?
1. In to paid up and reduced Sum Assured would be payable on maturity.
2. Return of Premium on maturity
3. Deduct the underpaid premium from Sum assured and payable on Maturity
4. Policy was subject to lien.
Q44. Married Woman's Property Act 1874 provides that a life
2
insurance Policy that has been taken out by:
1. Married Woman
2. Married Man
3. Married Son
4. Married Daughter
Q45.

2
Q46.
If there is no claim in a year than what will be the benefit to the customer as NO
CLAIM BONUS
1. Discount in next year premium
2. Increase in Sum Assured next year
3. Loans facility
4. No benefit
Q46. What is the maximum stake can foreign company has in one insurance company in
India? 2
1. 0.74
2. 0.26
3. 0.24
4. 0.76
Q47. Which is the Regulation that insists that all the insurance companies should provide
the information about the insurance ombudsman of that region while sending the 2
policy documents.
1. Policyholder grievance Regulation
2. Policyholder's Protection regulation
3. Ombudsman Regulation
4. Compliance Regulatory
Q48. A client demands the information on the current status of a
policy indicating accrued bonus. The insurer should provide 1
communication within
1. 10 Days.
2. 15 days.
3. 20 Days.
4. 30 Days.
Q49. Ethics can be defined as... 4
1. Those values we commonly hold to be "good" and "right".
2. Behavior that is based upon the moral judgments of an individual
3. A study of what makes one's own actions right or wrong.
4. All of the above.
Q50. The code of conduct has been prescribed in India by .......................... 1
1. IRDA
2. Insurance Council
3. Insurance Institute of India
4. Government of India.
Question
Q1. In term insurance if Critical illness rider claim happens then what will happen to Correct
existing policy Answer
1. CI benefit will cease 1
2. CI benefit reduced from existing sum assured
3. CI benefit continues
4. No change in policy.

Q2. The concept of indemnity is based on the key principle that policyholders should be 4
prevented from
1. Insuring existing losses.
2. Making false insurance claims.
3. Paying excessively for insurance cover.
4. Profiting from insurance.

Q3. A contract comes into existence when............................... 1


1. One party makes an offer which the other party accepts unconditionally.
2. One party makes an offer which the other party put extra conditions.
3. One party makes an offer where other party gives counter offer.
4. One party makes an offer which the other party receives the offer.

Q4. Suresh is suffering from Asthma and the policy is been done on
joint life basis and the need for nomination under the plan will be as 1
1. Joint life policy
2. Not Possible
3. 2.11805555555556
4. Not Applicable

Q5. Under the IRDA guidelines for Claim settlement, any queries or additional 3
documents can asked from the claimant within
1. 5 days from the receipt
2. 10 days from the receipt
3. 15 days from the receipt
4. 20 days from the receipt

Q6. which of the following refres to specific event which might cause a loss ... 2
1. Peril
2. Hazard
3. Physical hazards
4. Uncertinity

Q7. Principle of utmost good faith will operate in existing policy


1. Every time premium is paid 2
2. If the policy has lapsed and it has to be revived.
3. If the insured person falls sick and is admitted to hospital.
4. If the insured person changes his job.

Q8. In case of a term plan the maximum premium of the accidental rider can be.
1. 100% of basic premium.
2. 50% of basic premium.
3. 30% of basic premium.
4. 35% of basic premium.

3
Q9. The proposer can withdraw from the contract, if they disagree with the terms1 and
conditions of the Policy, within a 'free look-in period' of
1. 15 days from the date of receipt of the policy document
2. 20 days from the date of receipt of the policy document
3. 25 days from the date of receipt of the policy document
4. 30 days from the date of receipt of the policy document

Q10. What are the different types of Assignments? 2


1. Full Assignment and Partial Assignment
2. Conditional and Absolute Assignment.
3. Life Assignment and General Assignment.
4. Standard Assignment and Non-Standard Assignment.

Q11. In an insurance contract 'consideraton' means ................................ 3


1. proposal form
2. Advisors confidential report
3. premium
4. claim

Q12. Shamsher has a health insurance policy of ' 1, 00,000 individually and from his
company for ' 2, 00,000. He falls sick and got hospitalized. His hospital bill
ran to ' 50,000. He claimed this amount from his individual policy. 1
Also, he placed the request with his company for group policy claim,
which was rejected. The reason for rejection is.
1. Indemnity contract.
2. Value contract.
3. Deemed contract.
4. Rolling contract.

Q13. Rakesh has bought an Endowment, Money back, Term & Annuity Plan he 1
would like to avail Loan from:
1. Endowment Plan
2. Term Plan
3. Money Back Plan
4. Annuity Plan

Q14. Life Insurance also known as: 1


1. Value Contract.
2. Indemnity
3. Commercial Contract
4. Speculative

Q15. Parvesh bought a policy an endowment plan but after one year insurer. Found
he had Aorta Surgery .now which will apply by insurer: 1
1. Indisputability Clause (Section 45) apply
2. Principal of Indemnity apply
3. Lien Clause
4. Utmost good faith apply

Q16. For an insurance agent, a low persistency ratio means :


1. Loss of renewal commission
2. High client satisfaction
3. Higher reputation
4. More earning of First Year Commission
2
Q17.
Mr. Guptha is recently detected with lung cancer. He would like to ^
take an insurance. What is your suggestion?
1. Time of death is uncertain, so insurance can be given
2. Only lung is affected so health insurance can be given.
3. He can take insurance after submitting health certificate
4. Cannot give insurance for health reason
Q18. In which one of the following statement an agent's commission will ^ be disclosed to the
customer ?
1. Customer Statement of the product
2. Benefit illustration of unit linked product
3. Benefit illustration of endowment product
4. Customer data sheet with the product
Q19. Manish being a sole earning member of his family not insured
himself But looking for insurance for his son who is student .there 2
is possibility of
1. Physical Hazard
2. Moral Hazard
3. Medical
4. No underwriting
Q20. Gautam wants to purchase a Kisan Vikas Patra. What is the most suitable place to purchase
3
it
1. Bank
2. Insurance Company
3. Post Office
4. Share Market
Q21. The two basic elements of most life insurance plans are 3
1. Guaranteed Benefit and Non-guaranteed Benefit.
2. Interest Benefit and Bonus Benefit.
3. Death Benefit and Maturity Benefit.
4. Bonus Benefit and Tax Benefit.
2
Q22. If RBI increases the interest rates then what will be the effect on share prices.
1. Shares will be more attractive.
2. Shares will be less attractive.
3. Fixed deposits will be more attractive.
4. Fixed deposits will be less attractive.
Q23. What is the similarity between Recurring Deposits & cumulative deposits in a bank 1

1. Guarantees
2. Taxation
3. Tenure
4. Lock in periods
Q24. In which section of Policy document, Information about the location of the insurance
Ombudsman had written?
1. Operative clause
2. Attestation
3. Information statement
4. Endorsements
If a policy with premium of Rs 5000 has matured, how much will be deducted when the
Q25. maturity claim arises 1
1. Nil
2. 0.01
3. 0.05
4. 0.1

Q26. In which of the following Bank pays the Interest on the deposit's
fund on monthly /quarterly /half yearly/ yearly basis as chosen by 3
depositor fund:
1. Saving Deposit
2. Cumlative deposit
3. Traditional Deposit
4. Fixed Deposit
Q27. The regulations issued by the IRDA, require that the decision on
1
the proposal must be conveyed to the proposer within
1. 15 days of receiving the proposal
2. 20 days of receiving the proposal
3. 25 days of receiving the proposal
4. 30 days of receiving the proposal
Q28. If the license of an agent has been disqualified by a designated authority in 2010 then
the person can apply for a license in which

2
year.
1. 2013
2.2015
3.2017
4. 2020
Q29. In case of a term plan the maximum premium of the accidental rider 3
can be.
1. 100% of basic premium.
2. 50% of basic premium.
3. 30% of basic premium.
4. 35% of basic premium.

Q30. To avail the income tax benefit at investment stage, premium should be
maximum 2
1. 10% of S1.
2. 20% of SA.
3. 30% of SA.
4. 40% of SA.
Q31. Where one can approach in case of dispute? 2
1. IRDA
2. Consumer Forum
3. Distric Forum
4. National Forum

Health insurance policy holder takes treatment in a hospital which doesn't have
cashless facility. How the policy holder will get 1
benefited
Q32.
1. Need to spend and Get claim from the insurer
2. Need to add that hospital as TPA
3. Need not to claim
4. Need to switch to the another insurer
Q33. The younger the age of an individual. .their liabilities will be. 1
1. the lower
2. the higher
3. the smarter
4. the superior
Q34. Anand received post taxation 5% return on his fixed deposit in a bank. If his 2
net return is 3%, what can be the reason
1. Administrative charge
2. Inflation
3. Interest rate
4. Market Risk
Q35. The objective of Fact Finding is to 4
1. Gather Clients Information only.
2. Identify only the client's needs.
3. To provide solution of company's choice.
4. Both identify clients' needs & gather information.
Q36. What is the stipulated time frame within which an insurer is supposed to
respond after receiving any communication from its policyholders? 3
1. 24 hours
2. 1 day
3. 10 days
4. 30 days
Q37. When client declines the recommendation by the insurance adviser even after 2
resolving the concerns, the adviser should
1. Persuade the client to purchase the policy.
2. Should ask for reference who might be interested in financial planning.
3. Should never approach the client ever again.
4. Should do nothing.
Q38. What key benefit high persistency ratios have on insurance adviser? 1
1. High renewal income.
2. Less renewal income.
3. Fewer client bases.
4. Unsatisfied clients.

Q39. Amit & Rashmi are newly married. Both are working couple. They want to
invest their savings of 100,000 annually to build corpus to make down
payment for their house 5 years from now. An adviser sold than a unit link 2
product to meet their requirement. This may result in
1. Ruining the long term reputation of company.
2. Opportunity of new business for adviser.
3. Bad reputation of the individual adviser.
4. Financial burden on the couple.

Who is a regulator, supervisor and monetary authority of the financial


system in India?
1. IRDA
2. SEBI
3. RBI
4. SBI
Q40.
3
Vinay doesn't want to take insurance on himself. He feels that his family
will survive with the funds available in the bank and monthly rentals
received from village. This comes under Risk ___________________
1. Transfer
Q41. 2. Control
3. Retaining 3
4. Avoidance
Q42. Mr. sham had taken a term plan in 2008 . He died of cancer in 20
Investigation about his death revealed that he was deducted cancer in 1
200What will happen to his death claim?
1. Claim will be rejected
2. Claim will be settled as Cancer was not deducted in 2008
3. Claim will be settled as he died in 2010
4. Claim will be delayed
Q43. Under what kind of Rider the insurance company pays the treatment
cost in the event of hospitalization of the insured, subject to term & 3
Condition of the rider.
1. Accidental death benefit (ADB)
2. Critical illness (CI)
3. Hospital Care (HC)
4. Term Rider.
Q44. If a case is already before the consumer forum, then the ombudsman 3
should
1. Give a recommendation
2. Give a joint decision with the consumer forum
3. Dismiss the case
4. Give an award.
Q45. Issuance of a license to a person has been stipulated in 2
..................... Act
1. Section 43 of the Insurance Act 1939
2. Section 42 of the Insurance Act 1938
3. Section 12 of the Insurance Act 1922
4. Section 34 of the Insurance Act 1932
Q46. Which official body decides to increase the interest rates? 2
1. Central bank of India
2. Reserve bank of India
3. Reserve bank of country
4. Central bank of country
Q47. Pooling of insurance applies to 1
1. all types insurance
2. All types of insurance except Motor insurance
3. Only life insurance
4. Only Non-life insurance
Q48. For annuity plans, before receiving regular/periodic annuity payments, the
individual can make a lump sum withdrawal. This is known as
3
commutation. Up to what proportion of the accumulated fund can be
withdrawn?
1. The entire fund can be withdrawn
2. Only half of the fund can be withdrawn
3. Only one third of the fund can be withdrawn
4. Only one fourth of the fund can be withdrawn
Q49. Shankar, an adviser, sold a term insurance policy and unit-linked insurance
policy (ULIP) to Amar, the client, who is unmarried and has no dependent. 3
Consequently, Shankar's action can be termed as
1. Underselling of insurance policies.
2. Churning.
3. Overselling of insurance policies.
4. Ethical practice by an adviser.
Q50. For Insurance industry which association to take steps to Develop Education
and research in insurance?
1. Life Insurance Council 1
2. Life Insurance Corporation
3. Insurance institute of India
4. Insurance Institute of India
Question
Q1. The Ombudsman's powers are restricted to insurance contracts of what value?
1. 1Lakh
2. 10 Lakhs Correct
Answer
3. 15Lakhs
4. 20 Lakhs 4

Q2. Which of the following do not include the channel of indirect marketing?
1. individual agents
2. bancassurance
3. insurance brokers
4. through internet
3
Q3. Life insurance company determine the level of risk based on
1. Future expenses.
2. Claim experiences.
3. Present expenses.
4. Targeted bonus rates.
2
Q4. With pooling of risks an insurance company pools the premium collected from several
individuals to insure them against similar risks. At what circumstances will the 1
insurance companies pool the risk of a life insurance and health insurance together?
1. Under no circumstances
2. Under conditions of the reinsurer
3. As directed by actuary
4. As per company policy

Q5. What is the main objective of taking the life insurance policy? 4
1. Tax benefit
2. Savings
3. Investment
4. Protection

Q6. Grouping the similar risks by Insurance Company is called as.... 4


1. Grouping of Risk
2. Risk Grading
3. Risk Assessment
4. Pooling of Risk

Q7. In a life insurance policy it is later found that the person doesn't have insurable interest
4
then the contract is
1. expired
2. reviewed
3. Void
4. Valid
Q8. Life insurance is the most important for which age group
1. Young
2. Pre- retirement
3. Retirement
4. children

2
If we hold 100 units in gold ETF, It means that how much grams we have in physical
1. 5 or 10 grams
2. 10 or 15 grams
3. 15 or 25 grams
4. 50 or 100 grams

Mr.Akash filled the proposal form but before submitting to the company he discussed with
the agent that he is not sure whether he can pay for 15 years . This attitude affects which part
of the contract?
1. Consideration in the contract
2. Capacity to contract
3. Consensus ad idem
4. Offer and Acceptance in the contract

Mr. Raj has taken a policy of 15 year term. He has paid the premium for 9 years. But now
unfortunately he has lost his job. He is unable to pay the premium. What can be the best
solution from the following ?
1. Converting the policy to Term policy
2. Converting the policy to Paid up
3. Converting the policy to money back
4. Converting the policy to whole Life

Under what circumstances the surrender of a policy should be recommended by the agent?
1. When a client have been sold the right solution.
2. When a client is holding a product having good value for money.
3. When the policy has been mis-sold and do not match his needs.
4. When the client is enjoying good financial status.

What key event is most likely to make an insurance contract not a valid contract?
1. The circumstances are legitimate.
2. Representation of facts by the policyholders is true.
3. The life assured is major.
4. There is no insurable interest attached to the policy.

On foreclosure, if Death claim arises before the payment of the surrender value, the payment
would be payable to:
1. Nominee
2. Legal heir of life Assured
3. Debotrs
4. forfeited the Premium

Jyoti is submitting is copy of permanents account number card as age proof for buying an
money back plan .her age consider as a
1. Non Standard Age Proof
2. Standard Age proof
3. an Address proof
4. Proof of Income tax payer

With reference to the principle of indemnity a life insurance policy is a.


1. Insurance contract.
2. Indemnity contract.
3. Value contract.
4. Major life contract.
In which one of the following statement an agent's commission will be disclosed to the
customer ?
1. Customer Statement of the product
2. Benefit illustration of unit linked product
3. Benefit illustration of endowment product
4. Customer data sheet with the product
To explain the benefits of a product, the insurance adviser should
1. Provide the product brochure to the client.
2. Provide Benefit illustration documents to client.
3. Provide the terms and conditions document to the client.
4. Provide the website address to the client.

Ramesh bought an endowment plan for tern year he pays the same Amount in every year is
called:
1. Gross Premium
2. Level Premium
3. Risk Premium
4. Net Premium

Mr. Varun who won a multi chain company would like to take an insurance. What will be the
best option for him from the following ?
1. He can take Surety insurance
2. He can take Business Partner insurance
3. He can take Key man Insurance
4. He Can take company insurance

Mr. Vinu got a job recently, he can't afford to pay to pay the higher premium as of now but in
future once he settles down with his job he can pay higher premium. Which one will be the
best plan?
1. Convertible Endowment plan
2. Convertible Term plan
3. Convertible pure Endowment
4. Convertible money back plan

What is the similarity between Recurring Deposits & cumulative deposits in a bank
1. Guarantees
2. Taxation
3. Tenure
4. Lock in periods

If a person want to maintain emergency funds the best place is a bank or


1. Equity market
2. ULIP
3. Debt mutual fund
4. FD

What is the advantage of converting physical gold assets to gold ETFs.


1. Liquidity
2. More gold in value
3. Purity
4. More conversion value
A customer surrenders his policy on Feb 2010 As per Agents code of conduct, an agent can
get a new policy from this customer from Feb which year
1. 2011
2. 2012
3. 2013
4. 2014
Mr. Rao has Rs. 10, 00,000 cash with him. He would like to use this amount for his
daughter's marriage which is going to happen with in Nine months. He would like to get
some returns from this money in these 9 months period. What is the best option to park his
money?
1. Insurance
2. Shares
3. Fixed Deposit in Bank
4. Mutual Funds.
National insurance academy has the following main functions
1. Calculating premium
2. Interact with the government
3. Training
4. Mortality assumptions

The rider which is given by the Insurance company pays for the treatment costs in the event
of hospitalization of the insured person is called
1. Critical Illness Rider
2. Hospitalization Care Rider
3. Accidental Benefit Rider
4. Surgical Care Rider

Mr. Ramakant is 35 years old. He has bought retirement plan for 20 years. This type of
pension plan is known as ..................................................
1. immediate annuity
2. life annuity
3. deferred annuity
4. joint life annuity

Ajay bought a share for Rs.110 and he sold when it was Rs.630.What had happened to his
share?
1. Capital Appreciation
2. Capital Profit
3. Capital Benefit
4. Capital Variation

Suresh has adequate reserve capital with him and he wishes to protect his income, moreover
he feels that if he does not die then he would need the amount. What type of plan should he
opt for?
1. Term Insurance Plan
2. Endowment plan
3. Return of premium plan
4. Pension plan

As an adviser why is it essential for you to carry out the financial planning exercise with the
prospective clients?
1. Individuals understand their real needs and can prioritize them.
2. Individuals have same financial needs at different stages of the life cycle.
3. Individuals do not understands their real needs and cannot prioritize them.
4. Individuals' real and perceived needs are same.
What do you mean by Double Income Family?
1. When a person of the family is involved in double businesses or jobs.
2. When both the life partners are earning members of the family.
3. When the income of a family is just the double of income of another family.
4. When the members of a family do not work at all.

In which of the following an agent will collect the customer's Personal data, professional data
and financial data
1. Fact Finding Sheet
2. Advisor Confidential Sheet
3. Customer Data sheet
4. Personal and financial data sheet

During the fact finding session as an agent you should follow which one of the following
process?
1. Prioritize, Identify & Quantify Needs.
2. Identify, Quantify & Prioritize Needs.
3. Quantify, Identify & Prioritize Needs.
4. Prioritize, Quantify & Identify Needs.
Agent who is a licensed intermediary is actually is ?
1. A legal person to act on behalf of the re-insurer
2. B.A legal person to act on behalf of the insurer
3. C.A legal person to act on behalf of the contract
4. D. An authorized agent to act on behalf of the legal company

When client declines the recommendation by the insurance adviser even after resolving the
concerns, the adviser should
1. Persuade the client to purchase the policy.
2. Should ask for reference who might be interested in financial planning.
3. Should never approach the client ever again.
4. Should do nothing.

Amit & Rashmi are newly married. Both are working couple. They want to invest their
savings of 100,000 annually to build corpus to make down payment for their house 5 years
from now. An adviser sold than a unit link product to meet their requirement. This may result
in
1. Ruining the long term reputation of company.
2. Opportunity of new business for adviser.
3. Bad reputation of the individual adviser.
4. Financial burden on the couple.

Under which one of the following acts the maximum remuneration that can be given to
insurance agents described?
1. Income Tax Act
2. Indian Contract Act
3. Insurance Act 1938
4. IRDA Act 1999

For the customer whose claim has been refused by the company, the three places where
he/she should follow up are - Ombudsman, IRDA Customer Grievance Cell and
1. Consumer Forum
2. COPA
3. Sebi
4. Life Insurance Council
Q41. Vinay doesn't want to take insurance on himself. He feels that his family will survive
with the funds available in the bank and monthly rentals received from village. This
comes under Risk _________________________________
1. Transfer 3
2. Control
3. Retaining
4. Avoidance

Q42. Mr.Baskar had taken a Term plan for a sum assured of Rs. 7 lakhs. He also has an ADB
rider worth Rs. 4 lakhs. Unfortunately Baskar died in a car accident. How much will 3
be the death claim settlement?
1. Total 7 lakhs will be paid as death had taken place
2. Total 4 lakhs will be paid as death had happened due to car accident
3. Total 11 lakhs will be paid
4. Total 7 lakhs will be paid without any deduction.

Q43. Which one of the following report is not considered in case of Pankaj Bought a plan in 2
2009 & died in road accident in 2011 :
1. Policy first information report.
2. Advisor confidential report
3. Post Mortem Report.
4. Coroners Report

Q44. What is the major reason for conducting fact finding exercise? 1
1. Need analysis
2. Understand about company
3. Introduction of agent
4. Understanding the Customer

Q45. Issuance of a license to a person has been stipulated in ...................................... Act 2


1. Section 43 of the Insurance Act 1939
2. Section 42 of the Insurance Act 1938
3. Section 12 of the Insurance Act 1922
4. Section 34 of the Insurance Act 1932

Q46. Which of the following team represents the members of GBIC ? 1


1. Representatives from all insurance companies
2. Representatives from all government bodies
3. Representatives from IRDA
4. Representatives from Insurance institute of India

Q47. For annuity plans, before receiving regular/periodic annuity payments, the individual
can make a lump sum withdrawal. This is known as commutation. Up to what 3
proportion of the accumulated fund can be withdrawn?
1. The entire fund can be withdrawn
2. Only half of the fund can be withdrawn
3. Only one third of the fund can be withdrawn
4. Only one fourth of the fund can be withdrawn

Q48. How Many offices of Ombudsmen the Government body has set up to Resolving
insured customer's grievances.
1. 15
2. 12
3. 14
4. 16
2
Q49. Mr. Gayaram, Advisor who advised the customer to close the old policy and
take new one. But new policy does not have any extra benefits but it will 2
give more commission to Mr. Gayaram. This is called as ..................
1. Switching
2. Churning
3. Redirecting.
4. Shifting.

Q50. For Insurance industry which association to take steps to Develop


1
Education and research in insurance?
1. Life Insurance Council
2. Life Insurance Corporation
3. Insurance institute of India
4. Insurance Institute of India
Question
In life insurance business if a person is working in calculating premium rates of
insurance products, then he is mostly likely a member of
1. institutes of actuaries of India
2. insurance institute of India
3. Charted institute of insurance
4. Insurance institute of risk management

Human beings need life insurance because ................................


1. death is certain
2. death is uncertain
3. the timing of death is uncertain
4. Death is the solution.

In Insurance terms, pooling of risk is


1. Using the same pool for paying claims of car & life insurance.
2. Using different pool for paying claims of life insurance.
3. Using the same pool for paying claims of life & house insurance.
4. Using the same pool for paying claims of life insurance.

What is the maximum sum assured under a micro insurance


1. 10000
2. 25000
3. 50000
4. 100000

Mr. Shyam is having 9 year old child. Which product is not to be given priority?
1. Health plan
2. Child Plan
3. Life Insurance
4. Retirement Plan

Grouping the similar risks by Insurance Company is called as....


1. Grouping of Risk
2. Risk Grading
3. Risk Assessment
4. Pooling of Risk

Life insurance is the most important for which age group


1. Young
2. Pre- retirement
3. Retirement
4. children

The reduction in the benefit illustration shows what.


1. Charges.
2. Mortality.
3. Interest.
4. Inflation.

The proposer can withdraw from the contract, if they disagree with the terms and
conditions of the Policy, within a 'free look-in period' of
1. 15 days from the date of receipt of the policy document
2. 20 days from the date of receipt of the policy document
3. 25 days from the date of receipt of the policy document
4. 30 days from the date of receipt of the policy document
Q10. Ravi was expecting a claim amount of Rs. 12, 00,000 from insurer. But it
was rejected. He feels that it is repudiated on wrong reasons. Which 2
consumer forum can he approach?
1. National Commission
2. District Level
3. State Level
4. Mandal Level
Q11. What are the different types of Assignments?

2
1. Full Assignment and Partial Assignment
2. Conditional and Absolute Assignment.
3. Life Assignment and General Assignment.
4. Standard Assignment and Non-Standard Assignment.

Q12. Mr. Shanth has taken an endowment policy of 20 years. He has paid
premium for 10 years and now the policy is in force. At this point of time 2
can Shanth take loan?
1. Mr. Shanth will not be granted any loan
2. Mr. Shanth can take loan which should be certain percentage of the surrender
value of the policy.
3. There is no concept of loan in insurance policy
4. loans are allowed only in term plans

Q13. Mr.Karan who has to go to abroad for 6 months on an official work decides
to leave his car with his friend Mr. Jim. What will be the validity of the 3
insurable interest in this case ?
1. The insurable interest between the car and jim is valid for 6 months
2. The insurable interest between the car and karan is valid for 6 months
3. The insurable interest between the car and karan is valid until he owns it
4. The insurable interest between the car and Jim is valid until Karan's return

Q14. Rakesh has bought an Endowment, Money back, Term & Annuity Plan he would like to avail
Loan from:
1. Endowment Plan
2. Term Plan
3. Money Back Plan
4. Annuity Plan

Q15. Parvesh bought a policy an endowment plan but after one year insurer.
1
Found he had Aorta Surgery .now which will apply by insurer:
1. Indisputability Clause (Section 45) apply
2. Principal of Indemnity apply
3. Lien Clause
4. Utmost good faith apply

Q16. If both parents of proposer died in their thirties due to heart attack what is the kind of peril or
hazard the proposer has?
1. Insurable hazard
2. moral hazard
3. non insurable hazard
4. Physical hazard

Q17. When an illiterate person wants to have a policy ...................


.............................................................................................. 1
1. an impression of the left thumb is taken and third party has to attest it
2. an impression of the left thumb is taken and the advisor has to attest it
3. an impression of the left thumb is sufficient and need not be attested
4. A relative of the illiterate person has to sign on behalf of that illiterate person.
Q18. Lien is imposed on a policy when underwriter feels that ......................
1. the risk associated might increase
2. the risk associated might decrease 2
3. the risk associated might not be harmful
4. the risk associated might be general

Q19. Rakesh purchased a life insurance policy. While writing a proposal form he hide that he
practices mountaineering. Sadly he died in an accident while climbing Mount
Everest. The insurers rejected the claim.What is the reason for rejection?
1. Innocent misrepresentation.
2. Fraudulent misrepresentation.
3. Concealment.
4. Non-Disclosure.

Q20. Mr. Shanth has got a pure endowment policy for 30 years for the sum

^
assured of Rs. 75,000,00/-. It can be paid to him .....................
1. when he dies
2. when he survives the term
3. when he is hospitalized
4. when he loses his job

Q21. Manmohan has recently purchased a house worth ' 50, 00,000 on loan. 4
Which insurance product you as an adviser will suggest?
1. Endowment Plan.
2. Money Back Plan.
3. Whole Life Plan.
4. Term Plan.

Q22. What is the advantage of converting physical gold assets to gold ETFs.

1
1. Liquidity
2. More gold in value
3. Purity
4. More conversion value

Q23. In which section of Policy document, Information about the location of the ^ insurance
Ombudsman had written?
1. Operative clause
2. Attestation
3. Information statement
4. Endorsements

Q24. A customer surrenders his policy on Feb 2010 As per Agents code of
conduct, an agent can get a new policy from this customer from Feb which 1 year
1. 2011
2. 2012
3. 2013
4. 2014

Q25. If a policy with premium of Rs 5000 has matured, how much will be
1
deducted when the maturity claim arises
1. Nil
2. 0.01
3. 0.05
4. 0.1
Q26. Under current regulations what is the maximum stake that the Foreign ^
Partner in Insurance Company hold?
1. 0.48
2. 0.5
3. 0.6
4. 0.26
Q27. The Authority of COPA is limited to what amount at the district level. 3
1. 10,00,000.
2. 20,00,000.
3. 50,00,000
4. 1,00,00,000.

Q28. Which of the following is not a feature of WOP rider? 4


1. The rider waives future premiums in the event of the disability or death of the
policy holder.
2. This rider is ideal for helping to prevent a policy lapsing due to non-payment of
premiums due to death or disability
3. The main strength of WOP is the payment of full sum assured.
4. WOP rider can be offered with all plans

Q29. Mr. Ramakant is 35 years old. He has bought retirement plan for 20 years. ^ This type of
3
pension plan is known as ..................................................
1. immediate annuity
2. life annuity
3. deferred annuity
4. joint life annuity

Q30. The life expectancy in India is constantly improving and is well above 60
years. This also brings along challenges. These challenges can be covered 3
through
1. Equity.
2. Bank fixed deposits.
3. Health insurance plan.
4. Savings insurance plan.

Q31. Where one can approach in case of dispute? 2


1. IRDA
2. Consumer Forum
3. Distric Forum
4. National Forum

Q32. A Professional insurance market carries ................. 1


1. Need - Based Selling
2. Product - Based Selling
3. Commission - Based Selling
4. Company - Based Selling.

Q33. Need analysis involves identifying ...................


........................................................................ 1
1. Financial provision to meet predictable and unpredictable needs.
2. Capacity to pay the premiums in future.
3. Existing insurance plans.
4. Family and Employment details.
Q34. During fact finding, What will be the next step after Identifying clients

^
1
need
1. Quantify clients need
2. Priorities Clients Need
3. Recommending Product
4. Fill up the proposal form
Q35. Which type of questioning is very useful to gather information from clients? 2
1. Closed Ended Questions
2. Open Ended Questions.
3. Interrogative Questions.
4. Clarification Questions.

Q36. According to Insurance Brokers Association of India, what is the most appropriate relationship
between Insurer and Broker?
1. Insurance broker represent the client and the insurer remunerate the broker
2. The client represent the broker and the insurer remunerate the broker
3. Insurer represent both client and broker remuneration
4. Broker only service the client

Q37. What is the stipulated time frame within which an insurer is supposed to respond after receiving
any communication from its policyholders?
1. 24 hours
2. 1 day
3. 10 days
4. 30 days

Q38. Satish as an insurance advisor while recommending to his client Ramesh is not suitable policies
-
1. Satish should check the Ramesh 's commitment to the needs
2. Satish should outline the reasons for recommendation for a particular policy
3. Satish should check the acceptance or rejection of the recommendation.
4. Should advised him to surrender all the polices which are not buy from satish.

Q39. Why a building long term relationship with clients is necessary? 3


1. A satisfied client may be the source of other potential clients
2. Reviewing financial needs and plans are necessary with changes
3. Agent has the option to recommending highest commission at any time.
4. Introduction of a new insurance product or a change in a client's Circumstance

Q40. For tax Saving Someone wants to fixed deposit in bank. What duration is

^
required for it?
1. 3 years
2. 5 years
3. 7 years
4. 2 years

Q41. Vinay doesn't want to take insurance on himself. He feels that his family
will survive with the funds available in the bank and monthly rentals 3
received from village. This comes under Risk ____________
1. Transfer
2. Control
3. Retaining
4. Avoidance
Q42. A missing person will be presumed to be dead only after.... 3
1. Three years
2. Five years.
3. Seven years.
4. Nine years.

Q43. Which one of the following report is not considered in case of Pankaj Bought a 2
plan in 2009 & died in road accident in 2011 :
1. Policy first information report.
2. Advisor confidential report
3. Post Mortem Report.
4. Coroners Report

Q44. Married Woman's Property Act 1874 provides that a life insurance Policy that has 2
been taken out by:
1. Married Woman
2. Married Man
3. Married Son
4. Married Daughter

Q45. If there is no claim in a year than what will be the benefit to the customer as NO 1
CLAIM BONUS
1. Discount in next year premium
2. Increase in Sum Assured next year
3. Loans facility
4. No benefit

Q46. Mr.Raghav is an expert in fact finding .Mr.Raghav helps his brother in his job by
sharing the information of his clients after the fact finding he does to procure 3
insurance. What will be the outcome ?
1. Raghav will be promoted to the next level
2. Raghav will be terminated
3. Raghav License will be cancelled
4. Customer will complain to Ombudsman

Q47. If a policy holder buys a policy from the advisor and lodges a complaint, it should be 1
treated as :
1. Same for all policies sold by advisor
2. Same for all policies sold by the advisor except corporate clients
3. Only for policy for which complaint has been given
4. None applicable

Q48. What the name of department that is established by IRDA to deal with customer 3
complaints?
1. Customer Complaint Department (CCD)
2. Customer Grievance Department (CGD)
3. Consumer Affairs Department (CAD)
4. Consumer Protection Department (CPD)

Q49. License of an agent was withdrawn in June 2010 due to malpractice. He can reapply
for his license in
1. 2015
2. 2014 1
3. 2013
4. 2017
Q50. The code of conduct has been prescribed in India by ........................... 1
1. IRDA
2. Insurance Council
3. Insurance Institute of India
4. Government of India. 3

3
Correct
Question Answer
Q1. If a client wants to compare between all financial products then the best person he can 4
approach is
1. Individual agent
2. Corporate agent
3. Bank
4. Broker

Q2. Amit is looking for term insurance plan for protection of his family, he is advised to 2
approach to:
1. Property Insurance
2. Life Insurance
3. Health Insurance
4. Liabilty Insurance

Q3. Insured can contact to seek the resolution of grievances they have against insurer to IRDA 4
through:
1. Complaint@gov.ird1.in
2. insurancecomplaints@irda.gov.in
3. irdacomplaints@gov.in
4. Complaints@irda.gov.in

Q4. What is the maximum sum assured under a micro insurance 3


1. 10000
2. 25000
3. 50000
4. 100000

Q5. Under the IRDA guidelines for Claim settlement, any queries or additional documents can
asked from the claimant within 3
1. 5 days from the receipt
2. 10 days from the receipt
3. 15 days from the receipt
4. 20 days from the receipt

Q6. Which of the following can be an example of moral hazard? 3


1. a family history of heart disease
2. a person working in a chemical factory
3. a person consuming alcohol
4. A teacher working in a primary school.

Q7. If the annual premium for a plan is 32000 and a frequency loading of 4% is added in a
quarterly premium what is the amount that needs to be paid. 2
1. 8000
2. 8320
3. 9456
4. 9240
Q8. If a life insurance policy is issued with a lien, it will be mention in
1. Proviso
2. Schedule
2
3. Terms & conditions
4. Endorsement
Q9. If the employer has insurable interest in the life of an employee, what kind of policy is this?
1. Surety insurance 2
2. Keyman Insurance
3. Partnership Insurance
4. Debtor Insurance

Q10. In the sales illustration, the reduction of the actual benefit amount is mainly due to 2
deduction of
1. Commission
2. Charges
3. Non guaranteed benefits
4. Reversionary Bonus

Q11. The proposer can withdraw from the contract, if they disagree with the terms and 1
conditions of the Policy, within a 'free look-in period' of
1. 15 days from the date of receipt of the policy document
2. 20 days from the date of receipt of the policy document
3. 25 days from the date of receipt of the policy document
4. 30 days from the date of receipt of the policy document

Q12. Ravi was expecting a claim amount of Rs. 12, 00,000 from insurer. But it was rejected. He
feels that it is repudiated on wrong reasons. Which consumer forum can he approach? 2
1. National Commission
2. District Level
3. State Level
4. Mandal Level

Q13. Mr. Josh was filling the proposal form but as his mother was sitting beside him, even
though he drinks and smokes he ticked "NO" in smoking & drinking column of 2
proposal form. This indicates ?
1. He has breached the non-disclosure of the fact
2. He has breached the company by concealing the facts
3. He has breached the company by fraudulent information
4. He had done an innocent misrepresentation

Q14. A contract exists between insurer and proposer when 1


1. A proposal has been accepted by insurer.
2. A policy document has been stamped by insurer.
3. A policy document has been received by the policyholder.
4. An insurer has made another proposal.

Q15. Manish took a loan from Pankaj of Rs.10 lac, Pankaj transferred his
Insurance policy to Manish of Rs.10 lac as a security with the agreement Of when
Loan is fully paid, policy title will be revert back to name of Manish is called:
1. Loan assignment
2. Conditional Assignment
3. Loan Agreement
4. Absoulte Assignment 2
Law of Large number helps the insures to
1. Calculate the premium
2. Increase the profitability
3. Ascertain the death ratio
4. Declare the bonus

In a case, the underwriter felt that the risk associated with the person would decrease with time,
then he would accept the case with
1. A clause
2. A Lien
3. A Loading
4. Level Premium

The underwriter can get the required information about the proposer in ..........................
1. Proposal form.
2. renewal receipt
3. brochure
4. annual report

Which one of the following is not source of information about the


1. Proposal Form 2.Insurance agent
3. neighbor of proposer
4. Medical of examination report

Incase of presumption of death


1. Not necessary to pay premium until court decree
2. Necessary to pay premium until court decree
3. Claim not admissible
4. Depends on case to case

Low risk products give ..........


1. High Returns
2. Low Returns
3. Moderate Returns
4. Good Returns

In case of life insurance, Insurable interest must exist


1. At inception of policy
2. Not needed
3. At the time of claim
4. Any time during the contract

Rakesh wants to buy a policy primarily for Risk Cover but at the end of the term he wants to
get at least some return. Under which policy he will get these benefits
1. Endowment plan
2. ROP plan
3. Whole life plan
4. Ulip

Bank interest is accumulated


1. Monthly
2. Yearly
3. Quarterly
4. Once in 6 months
Q25. Mr. Rao has Rs. 10, 00,000 cash with him. He would like to use this amount for his daughter's
marriage which is going to happen with in Nine months.
He would like to get some returns from this money in these 9 months period. What is the best 3
option to park his money?
1. Insurance
2. Shares
3. Fixed Deposit in Bank
4. Mutual Funds.
Q26. Vinod being an insurance agent can offer assistance to his client Ramesh by: 2
1. Providing him emergency fund.
2. Matching the product with Ramesh's financial need.
3. Recommendation of product with highest return.
4. Discourse saving in a purposeful and need based manner.
Q27. Venu, aged, 32, had a Insurance cover of 10, 00,000/- . He was approached by an advisor who
made some analysis and told him that his Insurance need is higher than 10, 00,000 and 1
suggested that he surrenders the existing policy and buys a new one. This is an example of
1. Churning
2. Proposing
3. Underwriting
4. Switching
Q28.
Mr. Suresh purchase one pension plan and accumulated Rs. 9,00,000 amount in his pension 1
fund. He would like to utilize commutation benefit before taking pension. What is amount Mr.
Suresh can withdraw as commutation?
Rs. 3, 00,000/
1. Rs. 4, 50,000/
2. Rs. 90,000/
3. Rs, 4, 85, 468/Mr. Ranga has taken Critical Illness Rider. In which of the following
Q29. scenarios the insurance company will pay him
2
1. When Mr. Ranga dies due to critical illness
2. When Mr. Ranga is diagnosed a critical illness
3. When Mr. Ranga's family member will be in critical illness
4. When Mr. Ranga dies due to severe road accident.
Q30. Kajal is investing for the purpose of retirement at the time of vesting Age before receiving the
regular annuity she has option to withdraw/commute Upto: 2
1. 2/3rd of Accumulated Fund
2. 1/2th of Accumulated Fund
3. 1/3rd of Accumulated Fund
4. 1/4th of Accumulated Fund
Q31. In the context of financial planning, how is the difference between real needs and perceived 2
needs best described?
1. Real needs are financial needs and perceived needs are non-financial needs.
2. Real needs are actual needs and perceived needs are based on a client's thoughts and desires.
3. Real needs are identified by the insurance agent and perceived needs are identified by the
client.
4. Real needs are needs which satisfy an objective and perceived needs are needs which do not
satisfy an objective.
Perceived needs are those....
1. Short term needs.
2. Imagined to be important by Advisor.
3. Imagined to be important by client.
4. Long term needs.

Which are the two primary needs of any customer in any point of life?
1. Investment and retirement
2. Investment and Protection
3. Investment and savings
4. Investment and life needs

Which of the following is true regarding Family Floater Health Insurance Plan?
1. A Family Floater Plan is the same as a Individual Plan
2. Only self and spouse can be covered in this plan
3. Any number of people may be covered in this plan.
4. The insurance cover is shared amongst the family members in no fixed Proportion.

Which one of the following documents distinguishes between Guaranteed and non-guaranteed
benefits?
1. KYC documents
2. Benefit illustration documents
3. Fact-finding sheet
4. Financial details

Agent who is a licensed intermediary is actually is ?


1. A legal person to act on behalf of the re-insurer
2. B.A legal person to act on behalf of the insurer
3. C.A legal person to act on behalf of the contract
4. D. An authorized agent to act on behalf of the legal company

When client declines the recommendation by the insurance adviser even after resolving the
concerns, the adviser should
1. Persuade the client to purchase the policy.
2. Should ask for reference who might be interested in financial planning.
3. Should never approach the client ever again.
4. Should do nothing.

Under which one of the following acts the maximum remuneration that can be given to
insurance agents described?
1. Income Tax Act
2. Indian Contract Act
3. Insurance Act 1938
4. IRDA Act 1999

Why a building long term relationship with clients is necessary?


1. A satisfied client may be the source of other potential clients
2. Reviewing financial needs and plans are necessary with changes
3. Agent has the option to recommending highest commission at any time.
4. Introduction of a new insurance product or a change in a client's Circumstance

A claim was paid in a policy and it was advertised in the newspaper also.
This indicates that the policy was.
1. It was assigned.
2. It was paid up.
3. It was lost.
4. It was surrendered.
Q41. Mr. Chintamani has taken a policy from ABC Company on 2nd March 2010,
but unfortunately he died on 18th August 201 His death claim is considered

2
as ............
1. Normal death claim
2. Early death claim
3. Abnormal death claim
4. Unnatural death claim

Q42. The agents duties and responsibilities ends

4
1. When the clients policy is issued.
2. When the client's needs have been established.
3. When the nominee has changed.
Q43. 4. When the maturity/claim is settle4.

When a policy is lost, insurance company take utmost care while settling

2
maturity claims because
2

1. The claim may not be genuine.


2. The policy may be pledged.
3. The facts were misrepresented.
4. There is no nomination.
Q44. If a case is already before the consumer forum, then the ombudsman should 3
1. Give a recommendation
2. Give a joint decision with the consumer forum
3. Dismiss the case
4. Give an award.

Q45. If there is no claim in a year than what will be the benefit to the customer as NO CLAIM
BONUS
1

1. Discount in next year premium


2. Increase in Sum Assured next year
3. Loans facility
4. No benefit
Q46. One person wants to take term plan for 20 lakh and wants to take ADB rider, he is not sure of
how much sum assured he needs to take for ADB rider.

2
What is your suggestion?
1. Needs to take expert's suggestion
Q48. 2. Equal to base cover
3. Any Amount he can take if he is ready to pay premium
4. 50% of SA max.
Q47. Group Insurance can be taken in following relationship
Q41.
1
1. employee- employer
2. husband-wife
3. family members
4. society members

Harsh's policy matured, however he was paid only 25% of the sum insured in ^ spite of all his
premiums been paid on time. This indicates that his policy is a
1. Term plan.
2. Endowment plan.
3. Money back plan.
4. Wholelife plan.

Q49.
Q49. Mr. Gayaram, Advisor who advised the customer to close the old policy and
take new one. But new policy does not have any extra benefits but it will 2
give more commission to Mr. Gayaram. This is called as ...................
1. Switching
2. Churning
3. Redirecting.
4. Shifting.

Q50. Which is not unethical behavior in below statements? 3


1. Over selling of Insurance policies.
2. Under selling of Insurance policies.
3. Explaining all details of the policy to customer.
4. Churning.
Question
Human beings need life insurance because ................................
1. death is certain
2. death is uncertain
3. the timing of death is uncertain
4. Death is the solution.

life insurance the risk is determined on the basis of ...........................


1. future data
2. past data
3. statistical data
4. mathematical data

In Insurance terms, pooling of risk is


1. Using the same pool for paying claims of car & life insurance.
2. Using different pool for paying claims of life insurance.
3. Using the same pool for paying claims of life & house insurance.
4. Using the same pool for paying claims of life insurance.

Mr. Kunal used to participate in Car race. While taking up the Insurance policy he
disclosed this information. What kind of hazard does it refers to
1. Physical hazard
2. Fraudulent representation
3. Moral hazard
4. Peril

Law of large numbers is worked out by which of the following?


1. Pooling of risk
2. Maintaining insurable interest
3. With utmost good faith
4. Randomness

The function of Insurance works on ..............


1. Risk Transfer
2. Risk avoid
3. Risk retention.
4. All of the above.

If the sum assured remains the same, what will be impact of net premium if the age of
the policyholder increases
1. It rises
2. It falls
3. It remains constant
4. Gross premiums increases

In case of a term plan the maximum premium of the accidental rider can be.
1. 100% of basic premium.
2. 50% of basic premium.
3. 30% of basic premium.
4. 35% of basic premium.

In the sales illustration, the reduction of the actual benefit amount is mainly due to
deduction of
1. Commission
2. Charges
3. Non guaranteed benefits
4. Reversionary Bonus
Q10. Total annual premium Rs 32000 quarterly loading done 4 %. Hence actual ^
quarterly premium will be
1. 7680
2. 9320
3. 8320
4. 6600
Q11. What are the factors involved in calculating Surrender Value of the Policy? 4
1. Number of year's premium paid.
2. Number of premiums payable.
3. Sum Assured
4. All of the above.

Q12. Where do you not find insurable interest in the following options ............................. 4
1. surety-co surety
2. employee-employer
3. husband-wife
4. brother-sister

Q13. When is premium considered / deemed to be paid? 2


1. When insured writes a cheque in favor of insurer.
2. When cheque amount is deposited in insurer account.
3. When the cheque is deposited with insurer office.
4. When cheque is posted/couriered by the insure4.

Q14. Lalu Yadav is having a saving plan with 20 years tenure. He has paid 5 annual
premiums but due to financial crisis is unable to make future premium. His 4
policy
1. Acquires surrender value.
2. Contract comes to an end.
3. Moneys will be forfeited.
4. Acquires paid up value.

Q15. Life Insurance also known as:

1
1. Value Contract.
2. Indemnity
3. Commercial Contract
4. Speculative

Q16. For which of the following reasons, the underwriter should ask beyond agent's ^ confidential report.
1. Physical Hazard
2. Moral Hazard
3. SA is too high
4. Pure Risk

Q17. Kamal is willing to pay 60000/- per annum for his ULIP policy. What should be
2
the SA in case he wants to avail the tax benefits?
1. 1 lacs
2. 3 lacs
3. 2 lacs
4. 1.50 lacs
Q18. For an insurance agent, a low persistency ratio means : 2
1. Loss of renewal commission
2. High client satisfaction
3. Higher reputation
4. More earning of First Year Commission

Q19. The underwriter can get the required information about the proposer in ...........................
.......................................................................................................................................... 1
1. Proposal form.
2. renewal receipt
3. brochure
4. annual report

Q20. An insurance agent sold two policies to two different persons having the same
policy term. He declares the commission to each of them. The commission of

^
one of the policy is more than the other. What should be the reasons for this difference?
1. Risk profile of both the policyholder are different
2. Age of both the candidate are different
3. One has chosen Single premium policy and other has chosen regular premium policy
4. Both have chosen different kind of policies.

Q21. In the case of life insurance, insurable interest should exist

1
1. At the inception of the policy
2. At the time of a claim
3. At the time of every renewal premium payment
4. All the above occasion

Q22. Raunak earns 80,000 per month as salary. He has taken a House loan of Rs.
500000. What will be the maximum amount of EMI that can be charged by the 1 Bank to recover
the loan amount?
1. Rs. 32000 per month
2. Rs. 24000 per month
3. Rs. 40000 per month
4. Rs. 48000 per month

Q23. Varun wishes to pursue a career in insurance and wishes to be in a


department which calculates the level of premium. In which department

1
should he join?
1. Actuary
2. Underwriter
3. Claim Department
4. Accounts

Q24. Under current regulations what is the maximum stake that the Foreign Partner ^ in Insurance
Company hold?
1. 0.48
2. 0.5
3. 0.6
4. 0.26

Q25. In which of the following Bank pays the Interest on the deposit's fund on monthly
/quarterly /half yearly/ yearly basis as chosen by depositor fund:
1. Saving Deposit
2. Cumlative deposit 2
3. Traditional Deposit
4. Fixed Deposit

2
Q26. Mukesh buys shares at lower price and sold at higher price, the Difference between the two
prices is known as:
1. Dividend Income
3
2. Captial Appreciation
3. Bonus Share
4. Interest on share

Q27. A lump sum withdrawal allowed as commutation in pension plans is 1


1. 1/3rd of the accumulation fund
2. 1/4th of the accumulation fund
3. 1/2nd of the accumulation fund
4. Full withdrawal is allowed

Q28. Customer has opted for a 5 yrs guaranteed annuity option. What will happen to annuity, if the
customer survives for 5 years after the end of guarantee period? 2
1. paid up to 75 Yrs
2. annuity will be continued for next 5 Years
3. till he die
4. not remembered

Q29. If the agent recommends the client to terminate an endowment plan and take a whole life in order 3
to earn higher commission its termed as
1. Switching
2. Doing a financial planning
3. Churning
4. Fact Finding

Q30. Customer has to pay the amount in regular intervals to create purchase price or Pension Fund. 2
We call this phase as ..........................................
1. Collection Phase.
2. Accumulation Phase
3. Pension Phase.
4. Primary Phase.

Q31. Mrs. Sheela received some amount out of her husband's death. In such a situation what will be 3
her prime focus?
1. Savings
2. Insurance
3. Investment management
4. Planning for pension

Q32. A Professional insurance market carries .................. 1


1. Need - Based Selling
2. Product - Based Selling
3. Commission - Based Selling
4. Company - Based Selling.

Q33. Mr. Vinodh has lot of inconvenience to reach office every day. So he wants to buy an expensive
car when he will have sufficient fund. Here Mr. Vinodh's need is
1. Real need
2. Perceived need
3. Imaginary need
4. Important need
2
Q34. Anand received post taxation 5% return on his fixed deposit in a bank. If his
2
net return is 3%, what can be the reason
1. Administrative charge
2. Inflation
3. Interest rate
4. Market Risk

Q35. Insurance Protects which of the following? 3


1. The life of the person paying compensation
2. The risk retained person 's family
3. The financial goal of the insured
4. The life of the nominee

Q36. The consequences of these risks which will affect specific individuals or local
3
communities in nature is called as
1. Pure risk
2. Financial risk
3. Particular risk
4. Physical hazard

Q37. Agent who is a licensed intermediary is actually is ? 2


1. A legal person to act on behalf of the re-insurer
2. B.A legal person to act on behalf of the insurer
3. C.A legal person to act on behalf of the contract
4. D. An authorized agent to act on behalf of the legal company

Q38. When client declines the recommendation by the insurance adviser even after
2
resolving the concerns, the adviser should
1. Persuade the client to purchase the policy.
2. Should ask for reference who might be interested in financial planning.
3. Should never approach the client ever again.
4. Should do nothing.

Q39. Satish as an insurance advisor while recommending to his client Ramesh is not ^ suitable policies -
1. Satish should check the Ramesh 's commitment to the needs
2. Satish should outline the reasons for recommendation for a particular policy
3. Satish should check the acceptance or rejection of the recommendation.
4. Should advised him to surrender all the polices which are not buy from satish.

Q40. Who is a regulator, supervisor and monetary authority of the financial system
3
in India?
1. IRDA
2. SEBI
3. RBI
4. SBI
Q41. Pure risk is classified under 4
1. Economic risk
2. Speculative risk
3. Financial risk
4. Insurable risk

Q42. Who will take the initiation to settle the maturity claim process? 4
1. Client
2. IRDA
3. Advisor
4. Insurance company
Q43. Which is the right statement regarding claim enquiry?

1
1. The insurance company makes enquire only on death claims.
2. The insurance company makes enquire on maturity claims only.
3. Enquiry will be done on both Maturity and Death Claims.
4. Enquiry will be done if death happens before one year from policy inception date.
Q44. As per AML regulation, it allows cash premium not over than 4
1. Rs.20000
2. Rs.25000
3. Rs.5000
4. Rs.50000

Q45. Which official body decides to increase the interest rates? 2


1. Central bank of India
2. Reserve bank of India
3. Reserve bank of country
4. Central bank of country

Q46. Mrs. Hansa an agent, has planned to sell maximum products within a short
span of time by giving maximum rebates to the customer to complete the 3
contest target of the company.
1. It will benefit both the company and the customer
2. Reputation of the company will be high due to offers to the customer
3. Agent will be terminated
4. Agent will be qualified for the contest

Q47. What is the minimum Sum Assured allowed for Micro Insurance? 1
1. 5000
2. 10000
3. 15000
4. 20000

Q48. If policy holder is not satisfy with term and condition the policy which She/he has received, then
has option to return policy with in ....................................................?
1. 15 Days
2. 20 Days
3. 25 Days
4. 10 Days

Q49. After doing the need analysis of the client, the agent advised the client to opt
for TROP product. But the client refused. According to ethical business 1
practices what will the agent do ?
1. Enquire about the refusal from the client
2. Suggest an alternative plan
3. Pass on to the superior
4. Pass on to the other agent

Q50. Which is not unethical behavior in below statements? 3


1. Over selling of Insurance policies.
2. Under selling of Insurance policies.
3. Explaining all details of the policy to customer.
4. Churning.
Q1. How many days does the Free Look Period last?
1. 15 days
Correct
2. 30 Days Question Answer
3. 45 days
1
4. 60 Days

Q2. Insurance Market divided into


1. Endowment and Money Back Insurance
2. Life and General (non-life) Insurance
3. Government and Private Insurance Markets
2
4. Health and Saving Insurance Markets

Q3. Which of the following do not include the channel of indirect marketing?
1. individual agents
2. bancassurance
3. insurance brokers
4. through internet 3

Q4. Mr. Kunal used to participate in Car race. While taking up the Insurance policy he disclosed
this information. What kind of hazard does it refers to
1. Physical hazard
2. Fraudulent representation
3. Moral hazard 1
4. Peril

Q5. In case of life insurance, the insurable interest should exist


1. At the time of taking the policy
2. At the time of claim
3. At the time policy matures
4. At the time of taking the policy & claim 1

Q6. Under the IRDA guidelines for Claim settlement, any queries or additional documents can
asked from the claimant within
1. 5 days from the receipt
2. 10 days from the receipt
3. 15 days from the receipt
3
4. 20 days from the receipt

Q7. Mr. shailsh has an endowment policy with 30 years policy term, he has paid for eight years. The
sum assured Rs.8,00,000/- and accumulated bonus Rs.60,000/-. What is the paid up value? 3
1. 213333
2. 229333
3. 273333
4. 293333

Q8. The proposer can withdraw from the contract, if they disagree with the terms and conditions of the
Policy, within a 'free look-in period' of
1. 15 days from the date of receipt of the policy document
2. 20 days from the date of receipt of the policy document
3. 25 days from the date of receipt of the policy document
4. 30 days from the date of receipt of the policy document

1
Q9. What is the major reason for self employed to take insurance. 4
1. Save Tax
2. Fluctuating income
3. High Returns
4. Protection

Q10. Rajesh is 34 years old and having 2 sons, Vineeth and Sumith. The level of risk appetite, Rajesh
belongs is _____________________________ .
1. Middle Level
2. Top Level
3. Low Level
4. High Level

Q11. Principle of Indemnity denotes ................


.............................................................. 1
1. Insurance can not be used to make a profit
2. Insurance should not taken by high risk people.
3. Insurance can not taken by politicians.
4. All of the above.

Q12. Which of the following statements in correct in connection with assignment? 1


1. Assignee cannot make fresh nomination in the policy
2. The assignor need not be major at the time of assignment.
3. Section 45 of the insurance act speaks about assignment.
4. Conditional assignment and absolute assignment are one and the same.

Q13. Mr. Josh was filling the proposal form but as his mother was sitting beside
him, even though he drinks and smokes he ticked "NO" in smoking &

2
drinking column of proposal form. This indicates ?
1. He has breached the non-disclosure of the fact
2. He has breached the company by concealing the facts
3. He has breached the company by fraudulent information
4. He had done an innocent misrepresentation

Q14. Lalu Yadav is having a saving plan with 20 years tenure. He has paid 5
annual premiums but due to financial crisis is unable to make future

4
premium. His policy
1. Acquires surrender value.
2. Contract comes to an end.
3. Moneys will be forfeited.
4. Acquires paid up value.

Q15. Jyoti is submitting is copy of permanents account number card as age proof 2 for buying an money
2
back plan .her age consider as a
1. Non Standard Age Proof
2. Standard Age proof
3. an Address proof
4. Proof of Income tax payer

Q16. Vishal and sandeep applied for a health plan in XYZ Life Insurance Company.
Vishal is asked to undergo a medical checkup but Sandeep is not asked to do 2 so. What will be
most possible reason?
1. Sandeep has taken another policy from XYZ Life Insurance Company
2. Vishal is older than Sandeep
3. Sandeep is earning more then Vishal
4. Vishal is working in a MNC
If a contract is signed by a 15 years old boy, this contract will be
1. Null and void
2. Invalid
3. Voidable
4. Valid

What is the limit of tax benefit that can be availed of under Section 80C?
1. 50,000/
2. 15,000/
3. 10,000/
4. 100,000/-

Ramesh bought an endowment plan for tern year he pays the same Amount in every year is called:
1. Gross Premium
2. Level Premium
3. Risk Premium
4. Net Premium

What does MDRT Stand for?


1. Million Dollar Round Table
2. Major Double Rupees Tag
3. Major Dollar Round Tag
4. Mean Disposition Residence Time

Term Insurance Plan will give....


1. Only Death Benefit.
2. Only Maturity Benefit.
3. Only Bonus Benefit.
4. Only Tax Benefit.

What is the similarity between Recurring Deposits & cumulative deposits in a bank
1. Guarantees
2. Taxation
3. Tenure
4. Lock in periods

An investor holds a wide range of shares. If the Reserve Bank of India announces a series of
significant interest rate increases, the prices of these shares are most likely to
1. Become volatile
2. Decrease
3. Increase
4. Remain unchanged

In case of life insurance, Insurable interest must exist


1. At inception of policy
2. Not needed
3. At the time of claim
4. Any time during the contract

In which of the following Bank pays the Interest on the deposit's fund on monthly /quarterly /half
yearly/ yearly basis as chosen by depositor fund:
1. Saving Deposit
2. Cumlative deposit
3. Traditional Deposit
4. Fixed Deposit
Mukesh buys shares at lower price and sold at higher price, the Difference between the two prices is
known as:
1. Dividend Income
2. Captial Appreciation
3. Bonus Share
4. Interest on share
The Premium on all riders put together should not exceed
1. 10% of the premium on the base policy
2. 20% of the premium on the base policy
3. 30% of the premium on the base policy
4. 40% of the premium on the base policy

According to IRDA guidelines, how long does an insurance company have to complete its
investigation of a claim?
1. 30 days
2. 90 days
3. 120 days
4. 180 days

In Cumulative bank deposit the interest that in normally compounded on what basis.
1. Monthly
2. Quarterly
3. Half Yearly
4. Annually

Ajay bought a share for Rs.110 and he sold when it was Rs.630.What had happened to his share?
1. Capital Appreciation
2. Capital Profit
3. Capital Benefit
4. Capital Variation

What is the ceiling of tax exemption under 80 c.


1. 1 lakh
2. 1.5 lakhs
3. 2 lakhs
4. 3 lakhs

The concept of need based selling involves


1. Selling what company wish to sell.
2. Selling what adviser wish to sell.
3. Selling what customer requirement is.
4. Selling what IDRA wants company to sell.

What do you mean by Double Income Family?


1. When a person of the family is involved in double businesses or jobs.
2. When both the life partners are earning members of the family.
3. When the income of a family is just the double of income of another family.
4. When the members of a family do not work at all.

During fact finding, What will be the next step after Identifying clients need
1. Quantify clients need
2. Priorities Clients Need
3. Recommending Product
4. Fill up the proposal form
The objective of Fact Finding is to
1. Gather Clients Information only.
2. Identify only the client's needs.
3. To provide solution of company's choice.
4. Both identify clients' needs & gather information.

The consequences of these risks which will affect specific individuals or local communities in nature
is called as
1. Pure risk
2. Financial risk
3. Particular risk
4. Physical hazard
What is the factor which has influence on persistency?
1. Role of Agent
2. Product Design.
3. Policy Servicing
4. All of the above.

Agent who is a licensed intermediary is actually is ?


1. A legal person to act on behalf of the re-insurer
2. B.A legal person to act on behalf of the insurer
3. C.A legal person to act on behalf of the contract
4. D. An authorized agent to act on behalf of the legal company

After undertaking financial planning exercise, the prospective client said that he does not have funds
for investments. To resolve this query, which skill of an agent would be tested?
1. Objection handling skills.
2. Listening skills.
3. Communication skills.
4. Client information gathering skills.

A claim was paid in a policy and it was advertised in the newspaper also.
This indicates that the policy was.
1. It was assigned.
2. It was paid up.
3. It was lost.
4. It was surrendered.

For tax Saving Someone wants to fixed deposit in bank. What duration is required for it?
1. 3 years
2. 5 years
3. 7 years
4. 2 years

Mr.Baskar had taken a Term plan for a sum assured of Rs. 7 lakhs. He also has an ADB rider
worth Rs. 4 lakhs. Unfortunately Baskar died in a car accident. How much will be the death
claim settlement?
1. Total 7 lakhs will be paid as death had taken place
2. Total 4 lakhs will be paid as death had happened due to car accident
3. Total 11 lakhs will be paid
4. Total 7 lakhs will be paid without any deduction.
Which one of the following report is not considered in case of Pankaj Bought a plan in 2009 & died in
road accident in 2011 :
1. Policy first information report.
2. Advisor confidential report
3. Post Mortem Report.
4. Coroners Report

If a case is already before the consumer forum, then the ombudsman should
1. Give a recommendation
2. Give a joint decision with the consumer forum
3. Dismiss the case
4. Give an award.

Which of the following team represents the members of GBIC ?


1. Representatives from all insurance companies
2. Representatives from all government bodies
3. Representatives from IRDA
4. Representatives from Insurance institute of India
Mr.Raghav is an expert in fact finding .Mr.Raghav helps his brother in his job by sharing the
information of his clients after the fact finding he does to procure insurance. What will be the
outcome ?
1. Raghav will be promoted to the next level
2. Raghav will be terminated
3. Raghav License will be cancelled
4. Customer will complain to Ombudsman

If a policy holder buys a policy from the advisor and lodges a complaint, it should be treated as :
1. Same for all policies sold by advisor
2. Same for all policies sold by the advisor except corporate clients
3. Only for policy for which complaint has been given
4. None applicable

The charges were not fully disclosed to the customers” is a common complaint against
1. IRDA
2. Agents
3. Shareholders
4. Underwriters

Under the hospital care rider what is the payout made


1. 10% of the sum assured
2. Specified amount multiplied by the number of days the policyholder is hospitalized
3. expenses incurred per day multiplied by no. of days stay in the hospital
4. 100% of Sum Assured

For an insurance advisor churning is ........................ practice


1. good
2. bad
3. compulsory
4. appreciable
Q1. If insured gets the two advance payment in the 5yrs and in the maturity he gets rest of sum
assured. What type of policy it is Correct
1. Money back policy Question Answer
2. Convertible plan
3. Term plan 1
4. Endowment policy

Q2. life insurance the risk is determined on the basis of ............................ 2


1. future data
2. past data
3. statistical data
4. mathematical data

Q3. Amit is looking for term insurance plan for protection of his family, he is advised to approach 2
to:
1. Property Insurance
2. Life Insurance
3. Health Insurance
4. Liabilty Insurance

Q4. How are perils and hazards normally distinguished under term insurance policies? 2
1. Perils are medical factors which influence the risk of dying and hazards are lifestyle
activities which influence the risk of dying.
2. Perils are risks that policyholders will die before a specified date and hazards are factors
which could influence that risk.
3. Perils are factors which affect the risk being insured and hazards are the size of the risk
being insured.
4. Perils are factors which could influence an insured event occurring and hazards are the
actual events which will trigger a payout
Q5. Vijay received his policy bond on 11th June, 201 Due to some personal
problems he has decided to cancel the policy on 8th July, 201 Can he cancel or return the 1
policy?
1. No, as 15 days period is over
2. Yes, as it is within 1 year
3. No, as 20 days period is over
4. Yes, as it is within 3 months

Q6. which of the following refres to specific event which might cause a loss ... 2
1. Peril
2. Hazard
3. Physical hazards
4. Uncertinity

Q7. If the sum assured remains the same, what will be impact of net premium if the age of the
policyholder increases
1. It rises
2. It falls
3. It remains constant
4. Gross premiums increases

3
Q8. Both the parties to a contract must agree and understand the same thing and ^ in the same sense
which is called
1. Consideration.
2. Legality of an object.
3. Consensus ad idem.
4. Acceptance.

Q9. Under this situation may leads to breach of the duty of utmost good faith. 4
1. Non disclosure of material facts.
2. Concealment of a material fact
3. Fraudulent misrepresentation of facts.
4. All of the above.

Q10. Which of the following statements in correct in connection with assignment?

1
1. Assignee cannot make fresh nomination in the policy
2. The assignor need not be major at the time of assignment.
3. Section 45 of the insurance act speaks about assignment.
4. Conditional assignment and absolute assignment are one and the same.
Q11. Mr. Shanth has taken an endowment policy of 20 years. He has paid premium
for 10 years and now the policy is in force. At this point of time can Shanth 2
take loan?
1. Mr. Shanth will not be granted any loan
2. Mr. Shanth can take loan which should be certain percentage of the surrender
value of the policy.
3. There is no concept of loan in insurance policy
4. loans are allowed only in term plans

Q12. An insurance contract commences when 2


1. Quotation is signed by proposer.
2. First Premium Receipt is issued.
3. Proposal Form is signed.
4. Policy Document is received by policyholder.

Q13. On foreclosure, if Death claim arises before the payment of the surrender value, the payment
would be payable to:
1. Nominee
2. Legal heir of life Assured
3. Debotrs
4. forfeited the Premium

Q14. Which clause lays down the mutual obligation of the parties regarding, Payment of Premium by
Life assured & payment of Sum Assured by
1. Lien Clause
2. Opertaive Clause
3. Proviso Clause
4. Schedule of Policy

Q15. Jyoti is submitting is copy of permanents account number card as age proof for buying an money
back plan .her age consider as a
1. Non Standard Age Proof
2. Standard Age proof
3. an Address proof
4. Proof of Income tax payer
Q16. Mr. Guptha is recently detected with lung cancer. He would like to take an insurance. What is
your suggestion?
1. Time of death is uncertain, so insurance can be given
2. Only lung is affected so health insurance can be given.
3. He can take insurance after submitting health certificate
4. Cannot give insurance for health reason
Q17. When an underwriter may consider Moral Hazard?
1. An individual is proposing SA 15 times his annual income.
2. Insurance is taken out by an individual with dependents.
3. A nominee is not a dependent.
4. A medical checkup is carried out nearby place of residence.

Q18. Which one of the following is not source of information about the
1. Proposal Form
2. Insurance agent
3. neighbor of proposer
4. Medical of examination report

Q19. Manish being a sole earning member of his family not insured himself But looking for
insurance for his son who is student .there is possibility of
1. Physical Hazard
2. Moral Hazard
3. Medical
4. No underwriting

Q20. In the personal statement, Harish declares that he consumes alcohol twice every week. This is a
1. Moral hazard.
2. Moral peril.
3. Physical hazard.
4. Physical peril.

Q21. Payment/Investments in Kishan Vikash Patra under post office schemes is done
1. Regularly with no fixed term
2. Lump Sum with no fixed term
3. Lump Sum for fixed period of time
4. Regularly for fixed period of time

Q22. If RBI increases the interest rates then what will be the effect on share prices.
1. Shares will be more attractive.
2. Shares will be less attractive.
3. Fixed deposits will be more attractive.
4. Fixed deposits will be less attractive.

Q23. What is purpose of investing money in debt mutual fund?


1. Easy access
2. Fixed income
3. Tax Benefits
4. Liquidity

Q24. What is the advantage of converting physical gold assets to gold ETFs.
1. Liquidity
2. More gold in value
3. Purity
4. More conversion value
Q25. In which section of Policy document, Information about the location of the insurance Ombudsman
had written?
1. Operative clause
2. Attestation
3. Information statement
4. Endorsements

Q26. A customer surrenders his policy on Feb 2010 As per Agents code of conduct, an agent can get a
new policy from this customer from Feb which year
1. 2011
2. 2012
3. 2013 2
4. 2014
Q27. The Authority of COPA is limited to what amount at the district level. 3
1. 10,00,000.
2. 20,00,000.
3. 50,00,000
4. 1,00,00,000.

Q28. In 'Daily hospitalization cash benefit plan' ................. 3


1. all the expense incurred will be reimbursed by the insurance company
2. Some percentage of the expense will be reimbursed by the insurance company
3. a fixed amount on daily basis is paid by the insurance company
4. The insurance company will pay all the expense only after consulting hospital authority.

Q29. Which of the following is not a feature of WOP rider? 4


1. The rider waives future premiums in the event of the disability or death of the policy holder.
2. This rider is ideal for helping to prevent a policy lapsing due to non-payment of premiums due
to death or disability
3. The main strength of WOP is the payment of full sum assured.
4. WOP rider can be offered with all plans

Q30. Mr. Ramakant is 35 years old. He has bought retirement plan for 20 years.
3
This type of pension plan is known as ..............................
1. immediate annuity
2. life annuity
3. deferred annuity
4. joint life annuity

Q31. A Professional insurance market carries ................ 1


1. Need - Based Selling
2. Product - Based Selling
3. Commission - Based Selling
4. Company - Based Selling.

Q32. Mr. Shanth is in his early 20s and has just started earning. His risk appetite is expected to be
1. high
2. low
3. normal
4. Neutral.
Q33. The timing of death is uncertain, so when one should take life insurance?
1. At the time of uncertainty
2. At an early age
3. Anytime in life
4. At the time when Advisor takes the decision

Q34. What should an agent do in order to understand the mental state of client in respect to his
investments in saving products
1. Fact finding
2. Consulting the client's parents
3. Consulting the family of the client
4. Reviewing his existing investments

Q35. The objective of Fact Finding is to


1. Gather Clients Information only.
2. Identify only the client's needs.
3. To provide solution of company's choice.
4. Both identify clients' needs & gather information.

Q36. What is the stipulated time frame within which an insurer is supposed to respond after receiving
any communication from its policyholders?
1. 24 hours
2. 1 day
3. 10 days
4. 30 days

Q37. Which one of the following is not a benefit of persistency?


1. Helps the client in achieving goals
2. Reduction in costs
3. Increased client satisfaction
4. Decreased revenues

Q38. Satish as an insurance advisor while recommending to his client Ramesh is not suitable policies -
1. Satish should check the Ramesh 's commitment to the needs
2. Satish should outline the reasons for recommendation for a particular policy
3. Satish should check the acceptance or rejection of the recommendation.
4. Should advised him to surrender all the polices which are not buy from satish.

Q39. Why a building long term relationship with clients is necessary?


1. A satisfied client may be the source of other potential clients
2. Reviewing financial needs and plans are necessary with changes
3. Agent has the option to recommending highest commission at any time.
4. Introduction of a new insurance product or a change in a client's Circumstance

Q40. A claim was paid in a policy and it was advertised in the newspaper also. This indicates that the
policy was.
1. It was assigned.
2. It was paid up.
3. It was lost.
4. It was surrendered.

Q41. Which of the following falls under voidable contract?


1. Misrepresentation
2. Lack of insurable interest
3. Fraud
4. None of the above
Q42. Generally insurance companies do not hold the premium in case of a fraud or
misrepresentation. However, due to which of the following circumstances the 2 insurer can retain
the premium of the policyholder
1. Fraudulent claim
2. Indisputability clause
3. Redressal procedure
4. Pending decision from Ombudsman
Q43. In the process of settling maturity claims.... 2
1. the company will wait until the claimant comes to office to demand the claim
2. the process is initiated by the company well in advance of the maturity date
3. it is the responsibility of the claimant to approach the company
4. If the claimant does not come within a month the entire maturity amount will be forfeited.

Q44. Ashish is looking at different plans of insurance for protection at the lowest ^
premium. Which is the best plan for him?
1. Term plan
2. Endowment plan
3. Whole life plan
4. Money back plan.

Q45. Which body has created a call center for logging a complaint 2
1. Life Insurance Council
2. IRDA
3. Insurance Association
4. Insurance Institute of India

Q46. Mr.Raghav is an expert in fact finding .Mr.Raghav helps his brother in his job
by sharing the information of his clients after the fact finding he does to 3
procure insurance. What will be the outcome ?
1. Raghav will be promoted to the next level
2. Raghav will be terminated
3. Raghav License will be cancelled
4. Customer will complain to Ombudsman

Q47. If a policy holder buys a policy from the advisor and lodges a complaint, it
1
should be treated as :
1. Same for all policies sold by advisor
2. Same for all policies sold by the advisor except corporate clients
3. Only for policy for which complaint has been given
4. None applicable

Q48. AML Program of every insurer to include ______________________ , apart from


Procedure, Training and audit.
1. Inspection by IRDA
2. Audit by Finance Ministry
3. Appointment of Principal Compliance Officer
4. Inspection by Auditor General

Q49. Under the hospital care rider what is the payout made 2
1. 10% of the sum assured
2. Specified amount multiplied by the number of days the policyholder is hospitalized
3. expenses incurred per day multiplied by no. of days stay in the hospital
4. 100% of Sum Assured
Q50. The code of conduct has been prescribed in India by .......................... 1
1. IRDA
2. Insurance Council
3. Insurance Institute of India
4. Government of India.

S-ar putea să vă placă și